9 - Nephrology

Editors: Schrier, Robert W.

Title: Internal Medicine Casebook, The: Real Patients, Real Answers, 3rd Edition

Copyright 2007 Lippincott Williams & Wilkins

> Table of Contents > Chapter 9 - Nephrology

function show_scrollbar() {}

Chapter 9

Nephrology

Tomas Berl

Isaac Teitelbaum

Acute Renal Failure

  • Under what circumstances is serum creatinine a reasonable marker for glomerular filtration rate (GFR)? How is the creatinine clearance estimated from the serum creatinine?

  • What clinical findings most commonly suggest the presence of acute renal failure?

  • What processes need to be considered when attempting to ascertain the cause of acute renal failure?

  • What are the most common causes of acute renal failure in hospitalized patients and in outpatients?

  • P.381


  • What are the urinary findings that assist in differentiating prerenal azotemia from intrarenal acute renal failure?

  • What are the complications of acute renal failure?

Discussion

  • Under what circumstances is serum creatinine a reasonable marker for GFR? How is the creatinine clearance estimated from the serum creatinine?

    The serum creatinine is a reasonable marker for creatinine clearance and GFR only in the steady state, that is, when the serum creatinine is neither increasing nor decreasing. In the steady state, the creatinine clearance (CCr) may be estimated from the serum creatinine (SCr) by the Cockroft-Gault equation:

    Another equation derived as a result of the Modification of Diet in Renal Disease (MDRD) study has recently been validated as a more reliable predictor of GFR in some circumstances, such as chronic kidney diseases:

    A GFR calculator utilizing this equation may be found at the website, http://www.nephron.com/cgi-bin/MDRD.cgi, and is also available on many handheld devices.

  • What clinical findings most commonly suggest the presence of acute renal failure?

    A rise in the blood urea nitrogen (BUN) and serum creatinine levels and development of oliguria (<400 mL per day) are the common clinical findings that suggest the presence of acute renal failure. However, the absence of oliguria does not exclude acute renal failure because the process may also be nonoliguric. In fact, 20% to 30% of patients with acute renal failure are nonoliguric (>400 mL per day).

  • What processes need to be considered when attempting to ascertain the cause of acute renal failure?

    In patients with acute renal failure, prerenal, postrenal, and intrarenal processes need to be considered. The respective causes of prerenal and postrenal azotemia as well as intrinsic renal disease are listed in Tables 9-1,9-2,9-3.

  • What are the most common causes of acute renal failure in hospitalized patients and in outpatients?

    In hospitalized patients, the most common cause of acute renal failure (45%) is acute tubular necrosis, followed by prerenal azotemia and obstruction. Glomerulonephritis, vasculitis, interstitial nephritis, and atheroembolic

    P.382


    disease comprise most of the remaining causes. In contrast, acute renal failure in outpatients is most commonly due to prerenal azotemia (70%), followed by obstruction. Drug nephrotoxicity [e.g., aminoglycosides, angiotensin-converting enzyme (ACE) inhibitors, angiotensin receptor blockers (ARBs), and nonsteroidal antiinflammatory drugs (NSAIDs)] accounts for most of the remaining cases.

    Table 9-1 Causes of Prerenal Azotemia

    • Reduced extracellular and intravascular volume
      • Gastrointestinal losses (vomiting, diarrhea, nasogastric suction)
      • Dehydration
      • Burns
      • Hemorrhage
    • Reduced intravascular volume but increased extracellular volume
      • Cirrhosis
      • Nephrotic syndrome
      • Congestive heart failure cardiogenic shock
      • Third-space fluid accumulation (postoperative from abdominal surgery, severe pancreatitis, peritonitis)
    • Hemodynamically mediated acute renal failure
      • Anesthesia
      • Nonsteroidal antiinflammatory agents (due to renal prostaglandin inhibition)
      • Inhibitors of the renin-angiotensin system (due to a decrease in efferent arteriolar tone)
      • Hepatorenal syndrome
    • Vasoconstrictor agents
      • Calcineurin inhibitors
      • Contrast agents

  • What are the urinary findings that assist in differentiating prerenal azotemia from intrarenal acute renal failure?

    The urinary findings that can be used to help differentiate between prerenal azotemia and intrarenal acute renal failure are listed in Table 9-4.

  • What are the complications of acute renal failure?

    The various complications of acute renal failure are listed by category in Table 9-5.

Case

A 65-year-old diabetic woman presents to the emergency room with right upper quadrant pain that radiates around to the back, together with nausea, vomiting, anorexia, lightheadedness, and a diminished urine output during the last 24 hours. She has no previous history of renal dysfunction. Her temperature is 37.5 C (99.5 F); supine, her blood pressure is 110/70 mm Hg and pulse is 80 beats per minute; upright, her blood pressure is 85/60 mm Hg and pulse is 110 beats per minute. The physical examination findings are otherwise remarkable for the presence of decreased skin turgor, dry mucosal

P.383


membranes, flat neck veins, and absence of axillary sweat. Her lungs are clear and the cardiac findings are normal. There is exquisite right upper quadrant abdominal tenderness that worsens with inspiration, her stool is guaiac negative, and no edema is noted. Neurologic examination reveals nonfocal findings.

Table 9-2 Causes of Postrenal Azotemia

  • Urethral obstruction
    • Valves
    • Strictures
  • Bladder neck obstruction
    • Prostatic hypertrophy
    • Bladder carcinoma
    • Bladder infection
    • Functional
      • Autonomic neuropathy
      • a-Adrenergic blockers
  • Obstruction of ureters, bilateral
  • Unilateral obstruction in solitary kidney
    • Intraureteral
      • Sulfonamide, uric acid, acyclovir, antiretroviral agent crystals
      • Blood clots
      • Stones
      • Necrotizing papillitis
    • Extraureteral
      • Tumor of cervix, prostate, bladder
      • Endometriosis
      • Periureteral fibrosis
      • Accidental ureteral ligation
      • Pelvic abscess or hematoma

The following laboratory data are obtained: hematocrit, 50.2%; white blood cell count, 19,500/mm3 with 82% polymorphonuclear leukocytes, 16% band forms, and 2% lymphocytes; platelets, 312,000/mm3; sodium, 146 mEq/L; potassium, 4.1 mEq/L; chloride, 111 mEq/L; carbon dioxide, 22 mEq/L; glucose, 195 mg/dL; BUN, 35 mg/dL; creatinine, 1.6 mg/dL; total bilirubin, 1.8 mg/dL; alkaline phosphatase, 289 IU; and aspartate aminotransferase (AST), 35 U/L.

Urinalysis reveals a pH of 5, a specific gravity of 1.028; 1+ glucose, trace ketones, occasional nonpigmented granular casts, and no cellular casts or bacteria. The urine sodium level is 10 mEq/L and the urine creatinine level is 80 mg/dL.

Abdominal ultrasonography reveals the existence of gallstones and dilatation of the biliary tree. The kidneys measure 11 cm but exhibit no hydronephrosis or increased echogenicity.

While in the emergency room, the patient's fever spikes to 39 C (102.2 F), which is accompanied by 3 minutes of rigors and a decrease in blood pressure to

P.384


P.385


80/50 mm Hg. She is admitted to the hospital with a diagnosis of acute cholecystitis for the purpose of observation and eventual cholecystectomy. She is given gentamicin [2 mg/kg intravenously (IV)] and ampicillin (2 g IV every 6 hours). Her urine output over 12 hours is 100 mL. The next morning, the following laboratory values are reported: sodium, 140 mEq/L; potassium, 5 mEq/L; chloride, 100 mEq/L; carbon dioxide, 15 mEq/L; glucose, 130 mg/dL; BUN, 40 mg/dL; and creatinine, 2.5 mg/dL. Urinalysis now reveals a pH of 5 and a specific gravity of 1.010 with occasional renal tubular epithelial cells and a rare, muddy-brown granular cast. The urine sodium level is 80 mEq/L and the urine creatinine level is 40 mg/dL. Blood cultures are positive for a gram-negative bacillus.

Table 9-3 Causes of Intrarenal Acute Renal Failure

  • Glomerular diseases
    • Rapidly progressive glomerulonephritis
    • Postinfectious glomerulonephritis
    • Focal glomerulosclerosis associated with acquired immunodeficiency syndrome
  • Tubulointerstitial nephritis
    • Hypersensitivity reactions: penicillins, sulfonamides, fluoroquinolones, and many other drugs
    • Associated with systemic infections (Legionella, Toxoplasma)
  • Acute tubular necrosis
    • Ischemia, hypotension, septicemia
    • Direct drug toxicity: aminoglycosides, cisplatin, amphotericin, contrast agents
    • Myoglobin or hemoglobin
    • Acute tubular necrosis in pregnancy
  • Vascular diseases
    • Renal artery occlusion
    • Acute vasculitis
    • Malignant hypertension
    • Atheroembolic disease, multiple cholesterol emboli syndrome
    • Thrombotic microangiopathy
  • Others
    • Acute uric acid nephropathy
    • Hypercalcemic nephropathy

Table 9-4 Urine Findings in Prerenal Azotemia and Acute Renal Failure

Laboratory Test Prerenal Azotemia Intrarenal Acute Renal Failure
Urinary osmolality (mOsm/kg) >500 <400
Urinary sodium (mEq/L) <20 >40
Urine-plasma creatinine ratio >40 <20
Renal failure index: UNa/UCr/PCr <1 >2
Fractional excretion of sodium:
U Na/PNa UCr/PCr 100
<1 >2
Urinary sediment Normal or occasional granular casts Brown granular casts, cellular debris
UNa, urinary sodium level; UCr, urinary creatinine level; PCr, serum creatinine level; PNa, serum sodium level.
From Edelstein CH, Schrier RW. Acute renal failure: pathogenesis, diagnosis, and management. In:
Schrier RW, ed. Renal and electrolyte disorders, 6th ed. Philadelphia: Lippincott Williams & Wilkins
2003. Reprinted with permission.

Table 9-5 Complications of Acute Renal Failure

  • Metabolic
    • Hyponatremia, hyperkalemia, hypocalcemia, hyperphosphatemia, hypermagnesemia, hyperuricemia
  • Cardiovascular
    • Pulmonary edema, arrhythmias, hypertension, pericarditis
  • Neurologic
    • Asterixis, neuromuscular irritability, somnolence, coma, seizures
  • Hematologic
    • Anemia, coagulopathies, hemorrhagic diathesis
  • Gastrointestinal
    • Nausea, vomiting, bleeding
  • Infectious
    • Pneumonia, urinary tract infection, wound infection, septicemia
From Edelstein CH, Schrier RW. Acute renal failure: pathogenesis, diagnosis, and management. In: Schrier RW, ed. Renal and electrolyte disorders, 6th ed. Philadelphia: Lippincott Williams & Wilkins 2003. Reprinted with permission.

During the next 3 days, the patient remains oliguric and mild congestive heart failure develops. The BUN and creatinine levels rise steadily to 100 and 5.5 mg/dL, respectively.

  • At the time of arrival in the emergency room, what is the most likely explanation for this patient's acute renal dysfunction, and why?

  • At the time of the patient's arrival in the emergency room, what treatment would you prescribe, and why?

  • What is the cause of the continuing rise in the serum creatinine level after the patient is admitted to the hospital, and why?

  • What is the role for diuretics in this patient, and what is the proper dosage?

  • What is the appropriate approach to fluid management when the patient becomes oliguric?

  • What are the indications for acute dialysis in acute renal failure, and what alternative extracorporeal procedures could be considered?

P.386


Case Discussion

  • At the time of arrival in the emergency room, what is the most likely explanation for this patient's acute renal dysfunction, and why?

    There is no evidence for a postrenal cause of the acute renal failure in this patient, given the renal ultrasound study showing no obstruction. This leaves prerenal and intrarenal causes as the source of the acute renal failure. The history and physical examination findings suggest prerenal azotemia stemming from volume depletion. The laboratory data that corroborate this diagnosis include a BUN creatinine ratio that exceeds 20 and a fractional extraction of sodium (FENa) of 0.13%. The FENa is calculated as follows: UNa/PNa UCr/PCr 100% = 10/146 80/1.6 100% = 0.13%, where UNa and PNa are the urine and serum sodium levels, respectively, and UCr and PCr are the urine and serum levels of creatinine, respectively. In the setting of oliguria (<400 mL of urine per day), an FENa of less than 1% implies prerenal azotemia, whereas an FENa of greater than 2% implies an intrarenal process. In patients who are volume contracted due to diuretic use, the FENa is often elevated. In such patients the fractional excretion of urea (FEurea) may be more useful, calculated as Uurea/Purea UCr/PCr 100. A value of less than 35% suggests prerenal azotemia.

  • At the time of the patient's arrival in the emergency room, what treatment would you prescribe, and why?

    In this clinical setting, repletion of the extracellular fluid volume is the most critical element of therapy. This can be accomplished by the administration of either normal saline or lactated Ringer's solution; 250 to 500 mL can be given rapidly over 1 to 2 hours. These solutions, which are devoid of colloid, distribute in both intravascular and extravascular spaces. Fluid infusion should be continued until the blood pressure changes are no longer evident and a euvolemic state has been restored. This will also be accompanied by the reappearance of sodium in the urine. In the setting of prerenal azotemia, this maneuver should promptly return renal function to baseline.

  • What is the cause of the continuing rise in the serum creatinine level after the patient is admitted to the hospital, and why?

    After she is admitted to the hospital, the patient's clinical picture becomes more consistent with an intrarenal cause of acute renal failure, such as acute tubular necrosis. This is supported by the presence of tubular epithelial cells and brown granular casts in the urine. In addition, both the decrement in the UCr/PCr to 16 and the increase in the FENa to 3.57% strongly support this diagnosis. As to the cause of the intrarenal injury itself, gram-negative sepsis appears to be the most likely culprit. Aminoglycosides can also cause acute renal failure; however, this patient received only one dose of the antibiotic and, more commonly, the associated renal failure is nonoliguric. Ampicillin can cause acute interstitial nephritis, which has been reported for a number of antibiotics. The urinalysis would be expected to show white blood cells, red blood cells, white blood cell casts, and eosinophils.

  • What is the role for diuretics in this patient, and what is the proper dosage?

    Diuretics have been used in an attempt to convert oliguric patients with acute renal failure to a nonoliguric state, which is associated with a better outcome and simpler fluid management. Whether this conversion truly alters the prognosis has

    P.387


    not been settled. Diuretics can play a major role in the treatment of fluid overload that accompanies the patient's diminished urine output. Because loop diuretics need to reach the luminal membrane in this setting, very high doses are required (240 to 300 mg IV of furosemide or 8 to 12 mg IV of bumetanide). Doses higher than these have been used, but are not associated with an improved outcome and can cause permanent ototoxicity.

  • What is the appropriate approach to fluid management when the patient becomes oliguric?

    When a patient is oliguric (urine volume 400 mL), fluid restriction is needed and intake should not exceed 1 L because daily insensible losses are estimated to be between 500 and 700 mL. Likewise, sodium and potassium restriction is necessary. Therefore, the administration of 1 L of 0.5 N NaCl (i.e., approximately 75 mEq of sodium) without potassium supplementation is likely to prevent expansion of the extracellular fluid volume, hyponatremia, and hyperkalemia. If the episode of acute renal failure is more prolonged, nutritional support is also important.

  • What are the indications for acute dialysis in acute renal failure, and what alternative extracorporeal procedures could be considered?

    Dialysis is undertaken whenever any of the complications of acute renal failure ensue. These are listed in Table 9-5. Most commonly, dialysis is instituted for the management of fluid overload that is refractory to diuretic therapy, hyperkalemia that is resistant to therapy, or metabolic acidosis that cannot be adequately treated with bicarbonate. In oliguric, catabolic patients, dialysis has also been used to prevent rather than treat uremic symptoms, the so-called prophylactic dialysis. Continuous venovenous hemofiltration (CVVH) and continuous venovenous hemodialysis (HD) are alternatives to intermittent HD, and are being used increasingly.

Suggested Readings

Edelstein CL, Schrier RW. Acute renal failure: pathogenesis, diagnosis, and management. In: Schrier RW, ed. Renal and electrolyte disorders, 6th ed. Philadelphia: Lippincott Williams & Wilkins, 2003: 401.

Kieran N, Brady HR. Clinical evaluation, management, and outcome of acute renal failure. In: Johnson R, Feehally J, eds. Comprehensive clinical nephrology, 2nd ed. Mosby, 2003.

Metabolic Acidosis

  • What is the definition of metabolic acidosis?

  • What compensatory mechanism is triggered by metabolic acidosis?

  • How is the anion gap calculated, and how is it helpful in evaluating metabolic acidosis?

  • What are the causes of a metabolic acidosis with an increased anion gap, and what is the anion responsible for the increased anion gap?

  • How is the osmolar gap calculated, and how is this value useful in evaluating patients with a metabolic acidosis?

  • P.388


  • What are the causes of a metabolic acidosis with a normal anion gap?

  • What is urinary anion gap (UAG) and in what circumstances is it useful?

  • What is the difference between proximal and distal renal tubular acidosis (RTA), and how are these two forms of RTA differentiated?

Discussion

  • What is the definition of metabolic acidosis?

    Metabolic acidosis is a disorder that results from either the addition of hydrogen ion or the loss of bicarbonate, which, if unopposed, results in acidemia. However, metabolic acidosis is not defined either as a decrement in the serum bicarbonate level or as any given systemic arterial pH because, in the setting of mixed acid base disorders. The serum bicarbonate level or pH, or both, may be normal or even elevated despite the presence of metabolic acidosis.

  • What compensatory mechanism is triggered by metabolic acidosis?

    When metabolic acidosis develops, the decrease in pH activates carotid chemoreceptors and central nervous system receptors to stimulate ventilation. The increase in the minute ventilation lowers the partial pressure of carbon dioxide (Pco2), thereby returning the pH toward normal.

  • How is anion gap calculated, and how is it helpful in evaluating metabolic acidosis?

    Metabolic acidosis is broadly classified on the basis of the presence or absence of an increased anion gap. The anion gap (in millimoles per liter) is calculated using the following formula: plasma sodium - (plasma chloride + plasma bicarbonate). In most laboratories, a normal anion gap is considered to be 12 2 mmol/L. A normal anion gap metabolic acidosis results from either the addition of hydrochloric acid or the loss of bicarbonate with the concomitant retention of chloride. Because chloride is retained and is included in the calculation, the anion gap metabolic acidosis is maintained in the normal range. An increased anion gap results from the addition of an exogenous or endogenous acid. The anions produced by these acids are not measured and chloride is not retained. The anion gap increases because bicarbonate is consumed to buffer the organic acid. For example, organic anion + H+ + NaHCO3- H2O + CO2 + Na organic anion + organic acid. Because the organic anion is not measured or included in the calculation, the anion gap increases.

  • What are the causes of a metabolic acidosis with an increased anion gap, and what is the anion responsible for the increased anion gap?

    The various causes of metabolic acidosis with an increased anion gap are listed in Table 9-6.

  • How is the osmolar gap calculated, and how is this value useful in evaluating patients with a metabolic acidosis?

    The plasma osmolality is calculated using the following formula: Calculated osmolality = 2[Na] + [glucose]/18 + [BUN]/2.8 + [ethanol]/4.6. The osmolar gap is equal to the measured osmolality minus the calculated osmolality. A normal osmolar gap is less than 10 mOsm/kg. When the osmolar

    P.389


    gap is elevated in an acidemic patient, ethylene glycol or methanol intoxication must be strongly suspected.

    Table 9-6 Causes of Metabolic Acidosis with an Increased Anion Gap

    Cause Anion
    Increased acid production  
      Diabetic ketoacidosis BHB, AcAc
      Lactic acidosis Lactate, pyruvate
      Starvation
      Alcoholic ketoacidosis BHB > AcAc
      Nonketotic hyperosmolar coma
      Inborn errors of metabolism
    Ingestion of acid-generating toxic substances  
      Salicylate overdose (>30 mg/dL) Variety
      Methanol ingestion Formate, lactate
      Ethylene glycol ingestion Lactate, glycolate, oxalate
      Solvent inhalation
    Failure of acid excretion  
      Acute renal failure Variety, SO4, PO4
      Chronic renal failure
    BHB, betahydroxybutyrate; AcAc, acetoacetate.

  • What are the causes of a metabolic acidosis with a normal anion gap?

    The causes of metabolic acidosis with a normal anion gap are listed in Table 9-7.

  • What is UAG and in what circumstances is it useful?

    On occasion, the UAG may help distinguish gastrointestinal loss from renal loss of HCO3- as the cause of hyperchloremic metabolic acidosis:

    The UAG is an estimate of the urinary ammonium that is elevated in gastrointestinal HCO3- loss but low in distal RTA. UAG is a negative value if urine ammonium is high (as in diarrhea; average, -20 mEq/L), whereas it is positive if urine ammonium is low (as in distal RTA; average, +23 mEq/L).

  • What is the difference between proximal and distal RTA, and how are these two forms of RTA differentiated?

    RTA is one of the common causes of metabolic acidosis with a normal anion gap. Proximal RTA results from a failure to resorb the normal amount of bicarbonate in the proximal tubule, whereas distal RTA results from a defect in hydrogen ion secretion in the distal tubule. These two forms of RTA can be differentiated by determining the urine pH during systemic acidosis. In proximal RTA, when the serum bicarbonate, and therefore the filtered

    P.390


    bicarbonate level, is lowered to one that allows for proximal reabsorption of all the filtered bicarbonate, the urine can be maximally acidified (pH <5.4) as there is no increased distal delivery of unreabsorbed bicarbonate. In contrast, in distal RTA, the urine cannot be maximally acidified (pH >5.4) independent of the serum bicarbonate concentration.

Table 9-7 The Causes of a Metabolic Acidosis with a Normal Anion Gap

  • Gastrointestinal loss of HCO3
    • Diarrhea
    • Small bowel or pancreatic drainage or fistula
    • Ureterosigmoidostomy, long or obstructed ileal loop conduit
    • Anion exchange resins
    • Ingestion of CaCl2 or MgCl2
  • Renal loss of HCO3 -
    • Carbonic anhydrase inhibitors
    • Renal tubular acidosis
    • Hyperparathyroidism
    • Hypoaldosteronism
  • Miscellaneous
    • Recovery from ketoacidosis
    • Dilutional acidosis
    • Infusion of HCl or its congeners
    • Parenteral alimentation acidosisa
aSome formulas contain excess organic cations (balanced by Cl-), which yield H+ on metabolism.

Case

A 29-year-old man has been hospitalized in the psychiatry service for 2 months because of depression. The patient leaves the hospital on a pass and, on returning, complains of abdominal pain and vomiting. Over the next several hours, he becomes more agitated and is then found in an unarousable state and posturing.

Physical examination reveals a temperature of 102 F (38.8 C), pulse of 102 beats per minute, respiratory rate of 35 breaths per minute, and blood pressure of 160/100 mm Hg. The patient is unresponsive to pain. Funduscopic findings are within normal limits. No odors are noted on his breath.

Laboratory findings reveal the following: sodium, 142 mEq/L; potassium, 4.7 mEq/L; chloride, 111 mEq/L; bicarbonate, 10 mmol/L; serum calcium, 9.4 mg/dL; BUN, 12 mg/dL; and creatinine, 1.3 mg/dL. Arterial blood gas measurements performed on room air show a pH of 7.2, PCO2 of 17 mm Hg, and partial pressure of oxygen (PO2) of 100 mm Hg.

  • What is this patient's acid base disturbance, and what are the possible causes?

  • Why is the patient tachypneic, and is the compensation appropriate?

  • What other tests or laboratory findings would be useful in making the specific diagnosis?

  • P.391


  • In this patient, the serum glucose level proves to be normal and no serum ketones are detected. The plasma osmolality is 347 mOsm/kg and the osmolar gap is calculated to be 51 mOsm/kg. With the new information yielded by these additional tests, what possible diagnoses still remain?

  • How would you proceed to determine which substance is responsible for this patient's presentation?

  • How would you treat this patient?

Case Discussion

  • What is this patient's acid base disturbance, and what are the possible causes?

    The patient has an acidemia because the pH is 7.2. This could result from either a metabolic or a respiratory acidosis. The combination of a low PCO2 and a low serum bicarbonate concentration confirms the presence of a metabolic acidosis. In addition, the anion gap is elevated. The most likely causes of a metabolic acidosis with an increased anion gap, as outlined in Table 9-6, include diabetic ketoacidosis, lactic acidosis, starvation, alcoholic ketoacidosis, salicylate overdose, methanol or ethylene glycol ingestion, and renal failure.

  • Why is the patient tachypneic, and is the compensation appropriate?

    The patient is tachypneic as a compensatory response to the metabolic acidosis. If the patient were not tachypneic, the pH would be even lower and this would suggest an additional respiratory disorder. This patient is exhibiting an appropriate respiratory compensatory response. The serum bicarbonate level is decreased by 14 mmol/L from normal. Therefore, the PCO2 should be decreased by 14 to 21 mm Hg (Table 9-8). The patient has a PCO2 that is decreased by 21 mm Hg from normal, and this compensation is appropriate for the degree of metabolic acidosis involved. Table 9-8 summarizes the general expected compensatory responses to acid base disorders.

  • What other tests or laboratory findings would be useful in making the specific diagnosis?

    The patient clearly has a metabolic acidosis with an increased anion gap, but it is necessary to identify the specific cause with further testing. Initial tests that might elucidate the cause of the process include (a) the serum glucose level to determine whether hyperglycemia is present; (b) serum ketone levels to ascertain if acetoacetate is present; (c) serum salicylate and lactate levels to determine whether salicylate intoxication or lactic acidosis is present; and (d) serum osmolality to determine if the osmolar gap is elevated.

  • In this patient, the serum glucose level proves to be normal and no serum ketones are detected. The plasma osmolality is 347 mOsm/kg and the osmolar gap is calculated to be 51 mOsm/kg. With the new information yielded by these additional tests, what possible diagnoses still remain?

    With this additional information, you know that the patient has metabolic acidosis with an increased anion and osmolar gap. This limits the possible diagnoses to either methanol or ethylene glycol ingestion.

  • P.392


  • How would you proceed to determine which substance is responsible for this patient's presentation?

    Table 9-8 Rules of Thumb for Bedside Interpretation of Acid-Base Disorders

    • Metabolic acidosis
      • Paco 2 should fall by1.0-1.5 the fall in plasma [HCO3-]
    • Metabolic alkalosis
      • Paco 2 should rise by0.25-1.0 the rise in plasma [HCO3-]
    • Acute respiratory acidosis
      • Plasma [HCO3 -] should rise by approximately 1 mmol/L for each 10-mm Hg increment in Paco 2 ( 3 mmol/L)
    • Chronic respiratory acidosis
      • Plasma [HCO3 -] should rise by approximately 4 mmol/L for each 10-mm Hg increment in Paco 2 ( 4 mmol/L)
    • Acute respiratory alkalosis
      • Plasma [HCO3 -] should fall by approximately 1 - 3 mmol/L for each 10-mm Hg decrement in Paco2, but usually not to<18 mmol/L
    • Chronic respiratory alkalosis
      • Plasma [HCO3 -] should fall by approximately 2 - 5mmol/L per 10-mm Hg decrement in Paco2, but usually not to<14 mmol/L
    Paco2, arterial carbon dioxide tension; [HCO3 -], bicarbonate ion concentration.
    From Shapiro JI, Kaehny WD. Pathogenesis and management of metabolic acidosis and alkalosis. In: Schrier RW, ed. Renal and electrolyte disorders, 6th ed. Philadelphia: Lippincott Williams & Wilkins, 2003. Reprinted with permission.

    To determine which substance is responsible for this patient's presentation, both methanol and ethylene glycol levels should be assayed in the blood. In addition, the urine should be examined for the presence of calcium oxalate crystals, which are frequently present in the setting of ethylene glycol ingestion because of the metabolic conversion of the ethylene glycol to oxalate. In the setting of methanol intoxication, visual disturbances could ensue.

  • How would you treat this patient?

    The treatment of metabolic acidosis involves treating the underlying disorder. In acute metabolic acidosis, the rapid correction of pH through the administration of bicarbonate appears to produce derangements in cardiovascular function, probably caused by a paradoxical intracellular acidosis. The use of bicarbonate in this setting is therefore controversial. More specifically, two goals become important in a patient who has ingested ethylene glycol. The first is to inhibit the metabolism of ethylene glycol. Although ethylene glycol by itself is not a toxic substance, the metabolites produced by the liver are quite toxic and can precipitate acute renal failure and even cause death. Alcohol dehydrogenase (ADH) is the enzyme responsible for the metabolism of ethylene glycol, and it can be competitively inhibited by ethanol. Fomepizole, a direct inhibitor of ADH has also been employed. Ethylene glycol ingestion is still most commonly treated by the infusion of ethanol. The second goal

    P.393


    is to remove the ethylene glycol from the body. Ethylene glycol is excreted very slowly by the kidneys and, if the blood level is very high, HD may become necessary to improve removal of this substance from the blood. A similar approach is used for methanol ingestion.

Suggested Readings

Palmer BF, Alpern RJ. Normal acid-base balance and metabolic acidosis. In: Johnson R, Feehally J, eds. Comprehensive clinical nephrology, 2nd ed. Mosby, 2003.

Shapiro JI, Kaehny WD. Pathogenesis and management of metabolic acidosis and alkalosis. In: Schrier RW, ed. Renal and electrolyte disorders, 6th ed. Philadelphia: Lippincott Williams & Wilkins, 2003: 115.

Metabolic Alkalosis

  • What is the definition of metabolic alkalosis?

  • What are the processes involved in the generation of metabolic alkalosis?

  • What are the processes involved in the maintenance of metabolic alkalosis?

  • What are the two major categories of metabolic alkalosis, and what laboratory test is used to differentiate between the two?

  • What are the causes of NaCl-responsive metabolic alkalosis?

  • What are the causes of NaCl-resistant metabolic alkalosis?

  • What are the causes of metabolic alkalosis that are unclassified?

  • What is the compensatory mechanism that is stimulated by metabolic alkalosis?

Discussion

  • What is the definition of metabolic alkalosis?

    Metabolic alkalosis is a disorder that results from either the loss of hydrogen ions or the addition of bicarbonate, which, if unopposed, results in alkalemia. Metabolic alkalosis is not defined either as an increment in the serum bicarbonate concentration or as a given systemic arterial pH because, in the setting of mixed acid base disorders. The serum bicarbonate level or the pH, or both, could be either normal or even decreased in the presence of metabolic alkalosis.

  • What are the processes involved in the generation of metabolic alkalosis?

    Pathophysiologically, the development of metabolic alkalosis involves two phases (see Fig. 9-1). The first involves the generation of metabolic alkalosis. As follows from the definition just given, metabolic alkalosis can be generated as a result of either a net loss of hydrogen ions from the extracellular fluid, most commonly from either the upper gastrointestinal tract or more rarely through the kidneys, or from the net addition of bicarbonate or substances that generate bicarbonate (e.g., lactate, citrate, and acetate). In addition, the

    P.394


    loss of fluid having high concentrations of chloride and low concentrations of bicarbonate, as occurs with diuretic use and certain gastrointestinal tract diseases such as villous adenoma, generates a metabolic alkalosis.

    Figure 9-1 The factors responsible for the generation and maintenance of metabolic alkalosis. ECF, extracellular fluid; AII, angiotensin II; Paco2, partial pressure of carbon dioxide. (From

    Shapiro JI, Kaehny WD. Pathogenesis and management of metabolic acidosis and alkalosis. In: Schrier RW, ed. Renal and electrolyte disorders, 6th ed. Philadelphia: Lippincott Williams & Wilkins, 2003. Reprinted with permission.

    )

  • What are the processes involved in the maintenance of metabolic alkalosis?

    The kidney provides the corrective response to metabolic alkalosis by excreting excess bicarbonate. When the serum bicarbonate level exceeds 28 mEq/L, the anion appears in the urine, thereby preventing a further increase in its concentration. The maintenance of alkalosis therefore requires an alteration in renal bicarbonate reabsorption. Several factors constrain the kidney's ability to excrete bicarbonate and are important in the maintenance phase of metabolic alkalosis. Probably, the most important factor in this regard is extracellular fluid volume depletion, which serves to stimulate increased sodium resorption and bicarbonate reclamation in the proximal tubule. A decrement in GFR with a decrease in bicarbonate filtration contributes to the maintenance of the metabolic alkalosis. Another important factor in the maintenance of metabolic alkalosis is the chloride concentration. When the plasma bicarbonate concentration rises, the chloride concentration must fall. Because chloride is the only anion other than bicarbonate that can accompany sodium resorption, bicarbonate resorption is enhanced in its absence. Therefore, chloride must exist in sufficient quantity to allow for bicarbonate excretion. The hormone aldosterone stimulates the exchange of sodium reabsorption for hydrogen or potassium ion secretion in the distal tubule. With

    P.395


    the secretion of hydrogen ions, bicarbonate generation occurs in the plasma. Potassium ion depletion directly enhances bicarbonate reabsorption. An elevation in the Pco2 also stimulates bicarbonate reabsorption, and is important in the compensatory mechanism that keeps respiratory acidosis in check.

    Table 9-9 Causes of NaCl-Responsive Metabolic Alkalosis

    • Gastrointestinal disorders
      • Vomiting
      • Gastric drainage
      • Villous adenoma of the colon
      • Chloride diarrhea
    • Diuretic therapy
    • Correction of chronic hypercapnia
    • Cystic fibrosis

  • What are the two major categories of metabolic alkalosis, and what laboratory test is used to differentiate between the two?

    Metabolic alkalosis can be divided into two groups: NaCl responsive and NaCl resistant. The former is found in alkalemic patients who are volume depleted, and the latter in those with volume expansion. The most useful laboratory test for differentiating between the two groups is a spot urine chloride determination done before the initiation of therapy. In NaCl-responsive states, the urine chloride concentration is usually less than 20 mEq/L, and frequently even less than 10 mEq/L; in NaCl-resistant states, the urine chloride level exceeds 20 mEq/L. However, although metabolic alkalosis is routinely divided into these two categories, there are several disorders that are unclassified.

  • What are the causes of NaCl-responsive metabolic alkalosis?

    The causes of NaCl-responsive metabolic alkalosis are listed in Table 9-9.

  • What are the causes of NaCl-resistant metabolic alkalosis?

    The causes of NaCl-resistant metabolic alkalosis are listed in Table 9-10.

  • What are the causes of metabolic alkalosis that are unclassified?

    The unclassified causes of metabolic alkalosis are listed in Table 9-11.

  • What is the compensatory mechanism that is stimulated by metabolic alkalosis?

    When metabolic alkalosis develops, the alkalemia is sensed by chemoreceptors in the respiratory system. This leads to hypoventilation and an increase

    P.396


    in Pco2. As a general rule, the Pco2 (mm Hg) = 0.25 - 1.0 M [HCO3-] mEq/L, where Pco2 is the change in the Pco2. However, this hypoventilatory response is not as efficient as the hyperventilatory responses that accompany a metabolic acidosis.

Table 9-10 Causes of NaCl-Resistant Metabolic Alkalosis

  • Excess mineralocorticoid
    • Hyperaldosteronism
    • Cushing's
    • Bartter syndrome
    • Excessive licorice intake
  • Profound potassium depletion (800-1,000 mEq deficit)

Table 9-11 Unclassified Causes of Metabolic Alkalosis

  • Alkali administration
  • Recovery from organic acidosis
  • Antacids and exchange resins administered in renal failure
  • Milk-alkali syndrome
  • Massive blood or plasmanate (human plasma protein faction) transfusions
  • Nonparathyroid hypercalcemia
  • Glucose ingestion after starvation
  • Large doses of carbenicillin or penicillin

Case

A 25-year-old man with no previous medical history presents to the emergency room because of abdominal pain and severe vomiting of 2 days' duration, during which time he has been unable to eat or drink. He is taking no medications.

Physical examination reveals the following: temperature, 37.6 C (99.68 F); pulse, 120 beats per minute; respiratory rate, 18 breaths per minute; and blood pressure, 120/80 mm Hg. Orthostatic changes in the pulse and blood pressure are found, and there is mild, diffuse abdominal tenderness.

The following laboratory findings are reported: sodium, 140 mEq/L; potassium, 3.4 mEq/L; chloride, 90 mEq/L; bicarbonate, 35 mmol/L; and creatinine, 1.5 mg/dL. Arterial blood gas measurements on room air reveal a pH of 7.55, PCO2 of 44 mm Hg, and PO2 of 77 mm Hg.

  • What acid base disturbances are present in this patient?

  • What are the possible causes of this patient's metabolic alkalosis, and what laboratory test might be useful to elucidate the nature of the cause?

  • What factors are responsible for the generation and maintenance of the metabolic alkalosis in this patient?

  • If the patient's vomiting were to stop spontaneously, would the acid base disturbance also resolve?

  • How would you treat this patient?

Case Discussion

  • What acid base disturbances are present in this patient?

    The patient is alkalemic (pH, 7.55). Therefore, either a metabolic alkalosis or a respiratory alkalosis, or both, exist. The serum bicarbonate level is elevated to 35 mEq/L, and this indicates a metabolic alkalosis. In the setting of a respiratory

    P.397


    alkalosis, the PCO2 would be decreased, which is not the case in this patient. In the setting of metabolic alkalosis, the expected respiratory compensation (hypoventilation) would increase the PCO2. Because the PCO2 of 44 mm Hg is an increased value, this further supports the presence of a simple metabolic alkalosis with appropriate respiratory compensation.

  • What are the possible causes of this patient's metabolic alkalosis, and what laboratory test might be useful to elucidate the nature of the cause?

    As already discussed, metabolic alkalosis can be divided into two broad categories: NaCl-responsive and NaCl-resistant states. The hallmark of NaCl-responsive metabolic alkalosis is intravascular volume depletion. In this patient, the history of severe vomiting plus the vital signs that exhibit orthostatic changes are very suggestive of an NaCl-responsive metabolic alkalosis with intravascular volume depletion. The other causes of an NaCl-responsive metabolic alkalosis are nasogastric drainage, villous adenoma of the colon, chloride diarrhea, and diuretic therapy. Measurement of a spot urine chloride concentration would help confirm the diagnosis. In this patient, it would likely be low (<20 mEq/L).

  • What factors are responsible for the generation and maintenance of the metabolic alkalosis in this patient?

    In the metabolic alkalosis associated with vomiting, the loss of hydrogen ions in the vomitus is responsible for generating the alkalosis. Maintenance of the metabolic alkalosis is perpetuated by several factors. The NaCl lost with vomiting leads to a state of intravascular volume depletion, which, in turn, stimulates proximal tubule resorption of both NaCl and NaHCO3. It also stimulates the renin-angiotensin-aldosterone system. The resultant increased aldosterone secretion stimulates Na+/H+ and Na+/K+ exchange in the distal tubule. The former increases bicarbonate resorption, whereas the latter leads to potassium ion depletion, which also accelerates proximal bicarbonate resorption. The increased PCO2 associated with the compensation for metabolic alkalosis also increases bicarbonate resorption. These events are depicted in Fig. 9-1.

  • If the patient's vomiting were to stop spontaneously, would the acid base disturbance also resolve?

    Cessation of vomiting would not necessarily restore the acid base balance. The patient's vomiting is only the precipitating cause of his metabolic alkalosis. At this point, if his vomiting were to stop, several factors would still prevail (as discussed earlier) and maintain the metabolic alkalosis. Only when both the generating and maintaining factors are eliminated can the acid base disturbance resolve.

  • How would you treat this patient?

    In all cases, the treatment of metabolic alkalosis involves management of the underlying process. However, the process that has been the source of the metabolic alkalosis may have resolved, and other factors may be maintaining the metabolic alkalosis. Therefore, treating those factors that are maintaining the metabolic alkalosis may be most important. This patient should receive dual therapy. First, the vomiting (which is the source of the metabolic alkalosis) should be treated using an antiemetic agent. Second, the intravascular volume and potassium depletion must be corrected. This is accomplished by the administration of normal saline

    P.398


    plus supplemental potassium. The normal saline is administered until the orthostatic changes in the pulse and blood pressure resolve.

Suggested Readings

Gennari FJ. Metabolic alkalosis. In: Johnson R, Feehally J, eds. Comprehensive clinical nephrology, 2nd ed. Mosby, 2003.

Seldin D, Rector F. The generation and maintenance of metabolic alkalosis. Kidney Int 1972;1:306.

Shapiro JI, Kaehny WD. Pathogenesis and management of metabolic acidosis and alkalosis. In: Schrier RW, ed. Renal and electrolyte disorders, 6th ed. Philadelphia: Lippincott Williams & Wilkins, 2003:115.

Secondary Hypertension

  • What are the major causes of hypertension, and what is the nature of the pathophysiologic mechanism, or mechanisms, responsible for causing the elevation in blood pressure?

  • What should the initial evaluation of a patient who presents with an elevation in blood pressure consist of, and, based on the evaluation findings, what specific clinical features would point toward a particular secondary cause of hypertension?

  • If a secondary cause of hypertension is suspected, what would the further diagnostic evaluation comprise, and what would be the likely findings for each cause?

  • What are the respective treatment options for renal artery stenosis, pheochromocytoma, Cushing's syndrome, and primary hyperaldosteronism?

Discussion

  • What are the major causes of hypertension, and what is the nature of the pathophysiologic mechanism, or mechanisms, responsible for causing the elevation in blood pressure?

    Essential hypertension is the most common cause of hypertension and accounts for approximately 90% of all cases. It is usually asymptomatic. The usual age of onset is between 30 and 50 years and patients usually have a genetic predisposition for acquiring it. Other forms of hypertension must be ruled out by an initial screening evaluation before this diagnosis is confidently assigned. The regulation of arterial pressure involves a complex, and as yet not fully understood, interaction among neurohumoral mechanisms, sodium excretion, and baroreceptor reflexes. There is evidence to suggest that the mechanism responsible for the elevation of blood pressure in essential hypertension may involve inherited abnormalities in sodium excretion. This limitation in the

    P.399


    ability to excrete sodium may amplify the mechanisms that cause a rise in arterial pressure, thereby producing an abnormal response. These mechanisms include (a) an increment in the extracellular fluid volume and cardiac output, with secondary autoregulation causing an increment in peripheral vascular resistance; (b) an increase in the vascular response to vasoconstriction and (c) an increase in a putative circulating Na+/K+-adenosine triphosphatase inhibitor, which elevates the intracellular sodium and calcium levels, thereby also augmenting peripheral vascular resistance.

    Table 9-12 Identifiable Causes of Hypertension

    • Metabolic syndrome (obesity, insulin resistance, impaired glucose tolerance, dyslipidemia, hypertension)
    • Obstructive sleep apnea
    • Drug-induced hypertension
      • Decongestants
      • Adrenergic agents
      • Calcineurin inhibitors
      • NSAIDs
    • Chronic kidney disease
    • Primary hyperaldosteronism
    • Renovascular disease
    • Chronic steroid use or Cushing's
    • Pheochromocytoma
    • Coarctation of the aorta
    • Thyroid or parathyroid disease
    • NSAIDs, nonsteroidal antiinflammatory drugs.
    Modified from Nolan CR. The patient with hypertension. In: Schrier RW, ed. Manual of nephrology, 6th ed. Philadelphia: Lippincott Williams & Wilkins, 2005. Reprinted with permission.

    The major secondary causes of hypertension are listed in Table 9-12.

    The exact prevalence of renal artery stenosis is not known, but it probably accounts for approximately 5% of the general hypertensive population. It is an important diagnosis to make because it is the most common treatable form of secondary hypertension at any age, and it is one of the few potentially reversible causes of chronic renal failure. The diagnosis must be considered in any patient with severe hypertension refractory to therapy or in any patient who experiences the onset of hypertension either when very young or very old. Atherosclerotic plaques on the renal arteries are the cause in most cases, particularly in patients older than 50 years. Fibromuscular dysplasia, an entity seen in younger patients, particularly women, is the second most common cause of renovascular hypertension. There is evidence to suggest that both renin- and volume-dependent mechanisms play a role in the pathophysiology of renovascular hypertension in humans. The following evidence supports the interplay of both mechanisms: (a) the plasma renin activity is usually normal or high in patients with renal artery stenosis, but never low; (b) there is

    P.400


    unilateral hypersecretion of renin from the affected kidney with contralateral suppression; (c) in patients with unilateral renal artery stenosis, removal of the constriction or treatment with an inhibitor of the renin angiotensin system usually restores the blood pressure to normal or near-normal values; and (d) the effect of angiotensin blockade and salt restriction on blood pressure in patients with bilateral renal artery stenosis is frequently additive.

    Primary hyperaldosteronism is an uncommon cause of secondary hypertension, with a prevalence of approximately 1% in the hypertensive population. This disease can occur at any age. The classic form (Conn's syndrome) results from a unilateral adrenocortical adenoma, and accounts for approximately half the cases of hyperaldosteronism. The other half of the patients have bilateral adrenal hyperplasia. A small percentage has overproduction that can be suppressed with glucocorticoids. As in other forms of hypertension, the exact pathogenesis is unclear. The findings from early studies suggested that the expected salt and water retention secondary to the aldosterone excess raises the intravascular volume and subsequently cardiac output, thereby raising the blood pressure. However, hypervolemia is not a universal finding in patients with primary hyperaldosteronism. The results of studies in animals have suggested that the more important mechanism is an increase in sodium stores and total peripheral vascular resistance. The mechanism responsible for this is uncertain, but some study findings suggest that excess mineralocorticoids induce membrane changes in vascular smooth muscle, leading to abnormal cation turnover (possibly sodium and calcium), which, in turn, augments vasoconstriction and increases peripheral vascular resistance.

    Pheochromocytoma is also a rare cause of hypertension. It is estimated to affect 0.1% of patients with hypertension. Pheochromocytoma can occur at any age, but it arises most frequently in the fourth and fifth decades. In adults, most pheochromocytomas affect women. Pheochromocytomas are tumors of neuroectodermal origin. If they go undiagnosed, they carry a high risk of causing morbidity and mortality secondary to hypertensive crisis, shock, arrhythmias, cardiac arrest, and stroke. The hypertension of pheochromocytoma is a function of the norepinephrine released into the synaptic cleft. Circulating levels of norepinephrine have little direct involvement in the cause or maintenance of the hypertension.

    Hypertension complicates both acute and chronic renal parenchymal diseases, and affects approximately 80% to 90% of patients on dialysis. There are several mechanisms that may be involved in producing the hypertension in this setting, and these include (a) a markedly impaired ability of the diseased kidney to excrete salt and water; (b) the production of an unidentified vasopressor substance by the kidney; (c) absent production of a necessary humoral vasodilator substance by the kidney; (d) failure of the kidneys to inactivate circulating vasopressor substances; and (e) activation of the renin angiotensin system.

    The blood pressure in the upper extremities is elevated in 80% of children and adults with coarctation of the aorta. The mechanism responsible for this

    P.401


    hypertension is an inappropriate activation of the renin angiotensin system in the presence of an expanded body fluid volume.

    Hypertension affects 80% of patients with idiopathic Cushing's syndrome. Other clinical features of the disorder include glucose intolerance, menstrual disorders, sterility, loss of libido, acne, striae, osteoporosis, muscle weakness and wasting, edema, polyuria, and renal stones. However, the mechanism whereby adrenocorticotropic hormone and cortisol raise blood pressure in humans has not been elucidated, although there is evidence to suggest that glucocorticoids possess a hypertensinogenic action that is separate from their glucocorticoid activity.

    In the setting of renin-producing tumors, hypertension results from the excess secretion of renin by either a juxtaglomerular cell tumor or nephroblastoma. This causes the peripheral renin levels to be elevated, which mediates the hypertension.

  • What should the initial evaluation of a patient who presents with an elevation in blood pressure consist of, and, based on the evaluation findings, what specific clinical features would point toward a particular secondary cause of hypertension?

    The initial evaluation of patients with hypertension should include history taking, physical examination, and laboratory tests directed toward uncovering a correctable form of secondary hypertension.

    In terms of the history, a strong family history, as well as past observations of intermittent blood pressure elevations, suggest essential hypertension. Secondary hypertension often develops either before 30 or after 55 years of age. Other pertinent general questions should elicit information about steroid use, use of drugs, including oral contraceptives, and whether there have been recurrent urinary tract infections or a history of proteinuria, nocturia, trauma, or weight gain or loss.

    Physical examination should divulge further diagnostic clues as to the possible cause of the hypertension. The examination should focus on the patient's general appearance, muscular development, blood pressure and pulses in both upper extremities and a lower extremity, the supine and standing blood pressure, funduscopy, palpation and auscultation of the carotid arteries, cardiac and pulmonary examination, auscultation of the abdomen for bruits and palpation for an abdominal aneurysm and enlarged kidneys, and examination of the lower extremities for edema.

    Laboratory evaluation at the initial workup should include urinalysis for the presence of protein, blood, and glucose, together with a microscopic examination; the serum creatinine and BUN levels; hematocrit; the serum potassium level; the white blood cell count; the serum glucose, cholesterol, triglyceride, calcium, phosphate, and uric acid levels; electrocardiography; and a chest radiographic study.

    The clinical features that suggest renal vascular hypertension are listed in Table 9-13. The clinical features suggesting other secondary causes of hypertension are listed in Table 9-14.

    P.402


    Table 9-13 Clinical Features Suggestive of Renal Vascular Hypertension

    • Epidemiologic features
      • Hypertension in the absence of family history
      • Age <25 y or >55 y
      • Cigarette smoking
      • White race
    • Features of the hypertension
      • Abrupt onset of moderate to severe hypertension
      • Sudden onset of hypertension after abdominal trauma
      • Recent acceleration of severity of hypertension
      • Headaches
      • Resistance or failure of blood pressure control with usual therapy
      • Development of severe or malignant hypertension
      • Retinopathy out of proportion to severity of blood pressure
      • Excellent antihypertensive response to angiotensin-converting enzyme inhibitor
      • Deterioration in renal function in response to angiotensin-converting enzyme inhibitor
      • Blood pressure unaffected or increased with diuretic therapy
    • Associated features
      • Unprovoked hypokalemia
      • Hypokalemia in response to a thiazide diuretic
      • Abdominal or flank systolic-diastolic bruits
      • Carotid bruits or other evidence of large-vessel disease
      • Elevated peripheral plasma renin activity in absence of alternative explanation
    Modified from Ploth DW. Renovascular hypertension. In: Jacobson HR, Striker GE, Klahr S, eds. The principles and practice of nephrology. Philadelphia: BC Decker, 1991:379. Reprinted with permission.

  • If a secondary cause of hypertension is suspected, what would the further diagnostic evaluation comprise, and what would be the likely findings for each cause?

    A number of tests have evolved to assess the likelihood of renal vascular hypertension. Magnetic resonance angiography (MRA) or Doppler ultrasonography of the renal arteries have been used for the evaluation of renal artery stenosis. However, these tests have variable degrees of sensitivity and specificity, largely due to varying degrees of expertise with these techniques at different centers. Therefore, conventional renal arteriography remains the gold standard. It must be recognized, however, that the finding of renal artery stenosis provides no information concerning the pathophysiology of the vascular lesion. A postcaptopril (25 mg) elevation in plasma renin activity or a decrease in renal perfusion postcaptopril as assessed by scintillation techniques or renal vein renins can provide pathophysiologic information.

    If there are clinical features highly suggestive of a pheochromocytoma, the evaluation should begin with an assay of the total plasma catecholamine level, as measured through an indwelling 21-gauge butterfly needle in a patient who has been resting supine for 30 minutes. Values more than 2,000 pg/mL warrant performance of abdominal computed tomography (CT).

    P.403


    P.404


    Values between 1,000 and 2,000 pg/mL require performance of the clonidine suppression test to determine whether a pheochromocytoma is present. Clonidine does not suppress the release of catecholamines in patients with a pheochromocytoma, as it does in patients with essential hypertension. If the plasma catecholamine values are below 1,000 pg/mL, and the patient is hypertensive, the clonidine suppression test should be performed, but, if the patient is normotensive, the glucagon stimulation test may be helpful. For the glucagon test to be positive, the plasma catecholamine level must increase by threefold, or to greater than 2,000 pg/mL, 1 to 3 minutes after administration of the drug. If any of these test results are positive, abdominal CT should be performed. In patients whose clinical presentation suggests a pheochromocytoma but who have only a slight or moderate rise in the catecholamine level (<1,000 pg/mL), repeat testing, including measurement of the urinary catecholamine levels, should be performed.

    Table 9-14 Clinical Features of Other Secondary Causes of Hypertension

    • Primary hyperaldosteronism
      • History
        • Proximal muscle weakness, polyuria, nocturia, polydipsia, paresthesia, tetany, muscle paralysis, frontal headaches
      • Laboratory features
        • The diagnostic hallmark of this disease is hypokalemic metabolic alkalosis
        • Hyperglycemia may also be present
    • Pheochromocytoma
      • Symptoms
        • Patients may present in a wide variety of clinical settings, including transient ischemic attacks, stroke, headache (usually pounding and severe), palpitations with or without tachycardia, and excessive sweating; less common symptoms include tremor, pallor, nausea, weakness, fatigue, weight loss, and chest or abdominal pain
      • Physical examination
        • Postural hypotension occurs in 50%-75% of patients; paroxysmal episodes of hypertension occur in approximately one third of patients; sweating and muscular weakness may be evident
      • Laboratory features
        • Hyperglycemia or hypercalcemia may be present
    • Coarctation of the aorta
      • Symptoms
        • Epistaxis, throbbing headache, leg fatigue, cold extremities, and occasional claudication
      • Physical examination
        • Disparity in the pulsations and blood pressure between the arms and legs the pulsations in the upper extremities are pounding; those in the lower extremities are weak, delayed, or absent; the blood pressure in the arms exceeds that in the legs; there is collateral arterial circulation; murmurs are usually present but vary in location
      • Laboratory features
        • Chest radiograph may show prominence of the left ventricle, notching of the inferior border of the ribs from collateral vessels, and poststenotic dilatation of the aorta
    • Cushing's syndrome
      • Symptoms
        • Menstrual disorders, loss of libido, hirsutism, acne, striae, muscle weakness, easy bruising, edema, polyuria
      • Physical examination
        • Hirsutism, acne, striae, muscle weakness and wasting, purpura, bruising, edema, and poor wound healing
      • Laboratory features
        • Hyperglycemia, impaired glucose tolerance, neutrophilia, lymphopenia, and hypokalemia
    • Renal parenchymal disease
      • Symptoms
        • Uremia and anemia; associated with renal failure
      • Physical examination
        • If any findings, those associated with renal failure
      • Laboratory features
        • Several laboratory abnormalities may be present these include elevation of the BUN and creatinine levels, anemia, hypocalcemia, hyperphosphatemia, hyperkalemia, metabolic acidosis, proteinuria, and hematuria
    BUN, blood urea nitrogen.

    Echocardiography can visualize the area of aortic coarctation, but this is best confirmed by cardiac catheterization.

    Historically, Cushing's syndrome has been diagnosed on the basis of the following findings: elevated levels of urinary 17-hydroxycorticosteroids and urinary-free cortisol, loss of diurnal rhythm in the plasma cortisol concentrations, and failure of plasma cortisol levels to suppress overnight after a single 1-mg dose of dexamethasone. Because the overnight dexamethasone suppression test may not elicit suppression in obese and acromegalic patients, the low-dose dexamethasone suppression test (0.5 mg every 6 hours for 2 days) should be done to distinguish patients with Cushing's syndrome from healthy subjects. The high-dose dexamethasone suppression test (2 mg every 6 hours for 2 days) can distinguish Cushing's disease from an adrenal tumor, which does not suppress.

    If the cause of renal parenchymal disease cannot be identified with certainty on the basis of the history, physical examination, and laboratory findings, renal biopsy may be indicated. The biopsy results may shed light on whether the process is reversible, and thereby point toward treatment options, if any.

    P.405


    In the setting of renin-producing tumors, determination of the plasma renin activity by renal vein sampling usually shows a unilateral increase in the absence of a renal artery lesion.

  • What are the respective treatment options for renal artery stenosis, pheochromocytoma, Cushing's syndrome, and primary hyperaldosteronism?

    The treatment options for renal artery stenosis are either surgical or medical, and the choice depends on the patient involved. The surgical options include revascularization of the affected kidney using saphenous vein, autogenous artery, or synthetic (Dacron or polytetrafluoroethylene) grafts. A renal artery endarterectomy may be performed in patients with ostial atheromatous lesions. The most popular method of treatment, at least initially, is percutaneous transluminal balloon angioplasty with placement of stents. If these procedures are either unsuccessful or cannot be undertaken, medical management must be instituted.

    Cure of a pheochromocytoma consists of surgical removal of the tumor, and proper preoperative preparation helps reduce the attendant morbidity and mortality. In the presence of hypertension, administration of an adrenergic-blocking agent such as phenoxybenzamine (10 to 20 mg twice per day, increasing to 100 mg per day if tolerated) is recommended. Prazosin is not as effective. However, if the location of the tumor is in doubt or if multiple tumors are suspected, it is best not to administer -adrenergic blocking agents before surgery. The intravascular volume should be expanded both before and after surgery. In patients with inoperable malignant pheochromocytomas, drug therapy is needed. - and -Blockers may be used to control arrhythmias, or methyltyrosine may be prescribed to inhibit catecholamine synthesis.

    The best surgical approach in a patient with Cushing's disease is selected excision of the pituitary adenoma through a transsphenoidal approach. Surgical removal is sometimes followed by pituitary irradiation to prevent recurrence. A variety of drugs have also been used to treat patients with Cushing's disease. Adrenal tumors are best treated surgically.

    Hyperaldosteronism can be treated by either medical or surgical means. Mild aldosterone excess due to an adenoma, and all cases of bilateral hyperplasia, should be managed with aldosterone antagonists such as spironolactone because this disorder is not amenable to surgical treatment. Aldosterone-producing adenomas can be removed to effect cure once they have been appropriately localized by radiologic (CT) techniques.

Case

A 38-year-old adopted white man is seen by his family physician for the management of hypertension of 2 years' duration. Current medications include amiloride (5 mg) and hydrochlorothiazide (50 mg), with good blood pressure control until now. Review of systems reveals increasing fatigue, headaches, and muscle cramps. Physical examination reveals a blood pressure of 140/100 mm Hg in the left arm and 136/100 mm Hg in the right arm. No disparity in the blood pressure between the arms and the legs is found. The remainder of the examination findings are otherwise unremarkable.

P.406


The following laboratory data are reported: sodium, 145 mEq/L; potassium, 2.7 mEq/L; chloride, 109 mEq/L; bicarbonate, 29 mEq/L; BUN, 10 mEq/L; creatinine, 1.2 mg/dL; calcium, 9.1 mg/dL; cholesterol, 213 mg/dL; triglycerides, 163 mg/dL; uric acid, 6.1 mg/dL; phosphate, 2.1 mg/dL; and glucose, 99 mg/dL. Results of urinalysis, including microscopic examination, are normal.

The diuretics are stopped and the patient is placed on potassium supplements. Repeat laboratory work reveals that his sodium level is 147 mEq/L, potassium level is 3 mEq/L, and blood pressure is 146/104 mm Hg.

  • What is the differential diagnosis of this patient's hypertension?

  • What symptoms are related to the patient's hypokalemia?

  • What diagnostic steps would help confirm the diagnosis in this patient?

  • What are the treatment options in this patient?

Case Discussion

  • What is the differential diagnosis of this patient's hypertension?

    The differential diagnosis includes essential hypertension, primary hyperaldosteronism, pheochromocytoma, Cushing's syndrome, a renin-producing tumor, and renal artery stenosis. Renal parenchymal disease and coarctation of the aorta can be largely excluded as a cause of this patient's hypertension because the serum creatinine level and urinalysis findings are normal, as are the physical examination findings. The striking feature of this patient's hypertension is the hypokalemia despite treatment with a potassium-sparing diuretic plus potassium supplementation. Hypokalemia may be a feature of primary hyperaldosteronism, Cushing's syndrome, renal artery stenosis, and renin-producing tumors. Pheochromocytoma is considered a possibility because of the patient's complaints of headache and fatigue, although the clinical suspicion for this is low. Although hypokalemia occurs in Cushing's syndrome, the other clinical features of the disorder appear to be lacking. Renal artery stenosis is also unlikely unless the patient has fibromuscular dysplasia. Because the patient's family history is unknown, his genetic propensity for atherosclerosis is not known, but he does not appear to have other evidence of arteriosclerotic disease (e.g., bruits, angina, and claudication). Therefore, the most likely causes include primary aldosteronism and a renin-producing tumor. Essential hypertension can be diagnosed only after the most likely secondary causes have been excluded.

  • What symptoms are related to the patient's hypokalemia?

    Hypokalemia could explain this patient's headaches, muscle cramps, and fatigue. Additional symptoms may include muscle weakness, polyuria, and paresthesias.

  • What diagnostic steps would help confirm the diagnosis in this patient?

    Patients with a history of spontaneous hypokalemia, marked sensitivity to potassium-wasting diuretics, and refractory hypertension should be evaluated for primary hyperaldosteronism. The initial screening test is to determine the status of aldosterone excretion during prolonged salt loading. To perform this, 10 to 12 g of NaCl is added to the patient's daily intake. After 5 to 7 days of increased salt intake, the serum potassium concentrations and a 24-hour urine excretion of

    P.407


    sodium, potassium, and aldosterone are measured. The serum and urine potassium values indicate whether there is inappropriate kaliuresis (a serum potassium level of <3 mEq/L with a urine potassium level >30 mEq/24 hours). The 24-hour urine sodium level verifies compliance with the prescribed salt intake ( 250 mEq per day). If, under these conditions, the patient's rate of aldosterone excretion fails to show suppression below 14 g per 24 hours, this makes him a prime candidate for additional studies. The presence of hypokalemia and suppressed plasma renin activity further supports the diagnosis of primary hyperaldosteronism. This can be further confirmed by high aldosterone/renin ratio of greater than 100. If a renin-producing tumor were the cause of this patient's hypertension, the plasma renin activity would be elevated. If primary hyperaldosteronism is suspected, adrenal CT scanning should be performed. The finding of an adrenal mass would establish the diagnosis. Adrenal scintigraphy should be done if the CT findings are inconclusive. If the results of scintigraphy are also ambiguous, then adrenal vein sampling should be performed to measure the aldosterone levels. Adrenal vein sampling is still the most accurate test to localize aldosterone-producing tumors.

  • What are the treatment options in this patient?

    The hypertension associated with primary hyperaldosteronism can be managed adequately in most cases by means of salt and water depletion. The combination of spironolactone with hydrochlorothiazide or furosemide has been used successfully. However, if the adrenal adenoma is confined to one gland and there are no contraindications, the tumor should be removed. Only approximately half of patients are normotensive 5 years after surgery, but normal potassium homeostasis is restored permanently. If primary hyperaldosteronism stems from bilateral hyperplasia of the adrenal gland, this is best managed medically because surgical removal of too much of the adrenal gland can result in adrenal insufficiency.

Suggested Readings

Nolan CR. The patient with hypertension. In: Schrier RW, ed. Manual of nephrology, 6th ed. Philadelphia: Lippincott Williams & Wilkins, 2005:242.

Textor SC. Renovascular hypertension. In: Johnson R, Feehally J, eds. Comprehensive clinical nephrology, 2nd ed. Mosby, 2003.

Nephrolithiasis

  • What are the four major types of kidney stones, and which are radiopaque?

  • What is the shared pathogenesis for the formation of all types of kidney stones?

  • What are the fundamental causes of oversaturation of the urine?

  • What are the acute and chronic sequelae of kidney stones?

  • In the setting of uric acid kidney stones, is the oversaturation of urine with uric acid conditioned primarily by the urine pH or by the amount of uric acid excreted?

  • P.408


  • What are the three types of kidney stones that may present in the form of staghorn calculi, and what are the respective mechanisms responsible for their formation?

  • What are the principal causes of calcium stones?

  • What are the routine outpatient studies that should be performed in patients with recurrent stones?

  • What is the indication for measuring the excretion of uric acid in the setting of hypercalciuria?

  • What are the potential causes of hypercalciuria and how should it be treated?

Discussion

  • What are the four major types of kidney stones, and which are radiopaque?

    The principal types of kidney stones are composed of calcium salts, uric acid, cystine, and struvite. All except uric acid stones are radiopaque. Calcium-containing stones account for 80% of all stones, 15% are composed of struvite, 5% are made up of uric acid, and cystine stones are very rare.

  • What is the shared pathogenesis for the formation of all types of kidney stones?

    All kidney stones result from an excessive supersaturation of the urine. The ion concentration product at which salts in solution are in equilibrium with their solid phase is called the equilibrium solubility product. In the absence of a solid phase, salts may exist in a supersaturated state, above the equilibrium solubility product. In this setting, crystals composed of other compounds may act as heterogeneous seed nuclei that foster the formation of stones. If the ion product is sufficiently high, then new crystals form. Because an increase in urine volume leads to a decrease in the concentration of all solutes in the urine, an increased fluid intake of 2.5 to 3 L per day is part of the treatment for all kidney stones.

  • What are the fundamental causes of oversaturation of the urine?

    There are three major reasons for the oversaturation of urine: (a) hyperexcretion of a substance that is relatively insoluble in urine, (b) low urine volume, and (c) an abnormal urine pH. Citrate is a naturally occurring inhibitor of stone formation. Therefore, low urinary citrate excretion has also been implicated as an independent cause of calcium stone formation.

  • What are the acute and chronic sequelae of kidney stones?

    The acute consequences of kidney stones are urinary tract obstruction, infection, hematuria, pain, and, uncommonly, acute renal failure. Chronic consequences of nephrolithiasis are infection, RTA, and chronic renal insufficiency.

  • In the setting of uric acid kidney stones, is the oversaturation of urine with uric acid conditioned primarily by the urine pH or by the amount of uric acid excreted?

    Because monosodium urate is more soluble than uric acid, urate stones are rare. There is a significant risk for such stones only when the urinary form is mainly uric acid. Uric acid is a weak acid that has one proton that is dissociable under physiologic conditions with a pK (the negative logarithm of the ionization constant of an acid) of 5.3. Therefore, urate may exist in urine as

    P.409


    either monosodium urate or as uric acid. The concentration ratios of these two forms is a function of the ambient pH. A change in the urinary pH from 5 to 6.5 alters the undissociated acid concentration eightfold, whereas the urinary excretion of uric acid can increase only up to threefold. Therefore, changes in the urine pH play a greater role in uric acid stone formation than do changes in the amount of uric acid excreted.

  • What are the three types of kidney stones that may present in the form of staghorn calculi, and what are the respective mechanisms responsible for their formation?

    Uric acid, cystine, and struvite kidney stones may form in the renal collecting system and assume a staghorn configuration.

    Struvite kidney stones, which are the most common staghorn calculi, are a consequence of infection of the urinary tract with bacteria, usually Proteus species, which contain urease. This causes urea to be broken down to 2NH3 + H2O + CO2. Ammonia reacts with a proton, forming ammonium. This reaction raises the urine pH, resulting in an increased concentration of phosphate ions. These conditions spawn the formation of struvite (MgNH4PO4 6H2O), and may also lead to the formation of carbonate apatite (Ca10[PO4]6 CO3) crystals; therefore, struvite stones may contain variable proportions of carbonate apatite and struvite.

    Cystine stones are a manifestation of cystinuria, a rare hereditary disorder that is characterized by defects in dibasic amino acid transport. Normally, amino acids are almost completely reabsorbed by the proximal tubule. The urinary excretion of cystine is abnormally high in people with cystine stones, however, and this predisposes them to the formation of cystine stones. The urine pH has little effect on the solubility of cystine. The mechanism responsible for the formation of uric acid stones is discussed in the preceding question.

  • What are the principal causes of calcium stones?

    There are numerous specific causes of calcium kidney stones, but the major causes can be grouped into the following categories: low urinary volume, hypercalciuria, hyperoxaluria, hyperuricosuria, and alkaline urine. Hypocitraturia may also be an independent cause of calcium stone formation, although a low urinary excretion of citrate may actually be a consequence of an alkaline urine.

  • What are the routine outpatient studies that should be performed in patients with recurrent stones?

    The urine pH and volume should be assessed, and the 24-hour urinary excretion of sodium, calcium, uric acid, citrate, oxalate, phosphate, and creatinine should be determined.

  • What is the indication for measuring the excretion of uric acid in the setting of hypercalciuria?

    In the setting of hypercalciuria, uric acid crystals may act as seed crystals that initiate the precipitation of calcium oxalate from the urine. If patients are found to be hyperuricosuric, allopurinol treatment might be warranted.

  • What are the potential causes of hypercalciuria and how should it be treated?

    Most commonly, hypercalciuria is idiopathic in origin. Before making such a diagnosis, however, other causes of hypercalciuria (i.e., sarcoidosis,

    P.410


    immobilization, vitamin D excess, hyperthyroidism, Paget's disease, and malignant tumors with metastasis) need to be excluded (Table 9-15). The primary approach to treatment involves the attention to the underlying disorder when identified. For patients with idiopathic hypercalciuria, treatment is directed at lowering urinary calcium excretion. This is best achieved with thiazide diuretics, acting on the distal tubule. This approach needs to be coupled with a decrease in sodium intake, which will enhance proximal calcium reabsorption.

Table 9-15 Causes of Hypercalciuria

Cause Serum Calcium Level Other Serum Values Usual Stone Type
Idiopathic hypercalciuriaa  Normal Normal Calcium oxalate or calcium phosphate
Primary hyperparathyroidism High Hypophosphatemia, occasionally hyperchloremic acidosis Calcium oxalate or calcium phosphate
Renal tubular acidosis Normal Hyperchloremic acidosis Calcium phosphate  
aSarcoidosis, Cushing's mmobilization, vitamin D excess, hyperthyroidism, syndrome, alkali Paget's nd malignant tumors (which cause hypercalciuria, disease, rapidly progressive bone although not stones) must be excluded on clinical grounds.

Case

A 48-year-old man presents to a local emergency room because of right flank pain radiating to his right testicle that has lasted for 2 hours. The pain was initially mild and then became progressively severe over an hour. He has no nausea or vomiting, fever or chills, dysuria, hesitancy, or decreased urinary stream. He has no history of previous kidney stones or urinary tract infections. His past medical history is remarkable only for a history of Crohn's disease, which required resection of a portion of his ileum. He takes no medications.

On examination, he is found to be in obvious discomfort. His abdomen is soft and nontender with no masses. There is mild costovertebral angle tenderness. His testicles are normal. The remainder of his examination findings are unremarkable. The urine pH is 6, and urinalysis shows 1+ protein and 2+ heme. The sediment contains 10 to 15 red blood cells, 0 to 5 white blood cells per high-power field, and a moderate amount of amorphous crystals. There are no casts. His complete blood count and electrolyte levels are normal. A chest radiographic study and kidney, ureter, and bladder (KUB) film are interpreted as normal.

The following laboratory data are reported: calcium, 10 mg/dL; phosphorus, 3.7 mg/dL; albumin, 4.1 g/dL; creatinine, 1 mg/dL; and BUN, 12 mg/dL. His blood pressure is 140/85 mm Hg, pulse is 95 beats per minute, respiratory rate is 20 breaths per minute, and temperature is 37.2 C (98.96 F).

  • What are some of the possible renal causes of this patient's symptoms?

  • What is the significance of the crystalluria?

  • P.411


  • Does the absence of a colic-like pain suggest that this patient's pain is not due to a kidney stone?

  • What would be the appropriate test for confirming the diagnosis of a kidney stone in this patient?

  • Once the diagnosis of a kidney stone is established, what is the appropriate management that should be implemented in the emergency room?

    Noncontrast helical CT scan reveals a radiopaque stone at the left ureteropelvic junction. Subsequently, the patient passes the stone in his urine while in the emergency room. Laboratory analysis reveals that the stone is composed primarily of calcium oxalate. Subsequently, a 24-hour urine collection revealed an increase in urinary oxalate excretion (>50 mg per 24 hours).

  • What are the possible causes and the treatments of hyperoxaluria as seen in this patient?

Case Discussion

  • What are some of the possible renal causes of this patient's symptoms?

    Kidney stones, renal infarction, and papillary necrosis may all present with the acute onset of flank pain together with hematuria. However, renal infarction usually occurs in a patient who has either a local or systemic cause for thrombosis (e.g., trauma, aneurysm, or vasculitis involving the renal artery) or thromboembolism (e.g., endocarditis, mural thrombi, or fat emboli). Papillary necrosis typically occurs in patients with either advanced diabetic nephropathy or sickle cell disease. In contrast, kidney stones often arise in people who have no known contributory medical illness.

  • What is the significance of the crystalluria?

    Except for the finding of cystine crystals, which indicates cystinuria, crystalluria is of no diagnostic value when evaluating a patient for nephrolithiasis, as crystals can appear in normal urine.

  • Does the absence of a colic-like pain suggest that this patient's pain is not due to a kidney stone?

    No. Typically, the pain associated with kidney stones is a steady pain that gradually worsens; it does not fluctuate, as the term renal colic suggests.

  • What would be the appropriate test for confirming the diagnosis of a kidney stone in this patient?

    Although in some cases nephrolithiasis can be diagnosed on the basis of the KUB radiographic findings, it is usually necessary to perform excretory urography, as in this patient, to establish the diagnosis. It allows the location, size, shape, and radiolucency of kidney stones to be determined. Although retrograde pyelography can yield the same information, it is a more expensive and invasive procedure. Ultrasonography is not as sensitive as excretory urography for detecting kidney stones. More recently, noncontrast helical CT has become the procedure of choice in most centers.

  • Once the diagnosis of a kidney stone is established, what is the appropriate management that should be implemented in the emergency room?

    The patient should be kept well hydrated, usually with intravenous fluids, to maintain a brisk urine flow, which may promote passage of the stone, and to

    P.412


    diminish the risk of nephrotoxicity from the radiocontrast agent. All of the patient's urine should be strained to determine if the patient has passed any stones. If any stones are obtained, they should be sent for analysis. Patients almost always require narcotic analgesics for management of the pain. Patients should be admitted to the hospital if inadequate pain relief is obtained with oral analgesics, or in the event of urinary tract infection or acute renal failure.

  • What are the possible causes and the treatments of hyperoxaluria as seen in this patient?

    Hyperoxaluria can result in sufficient supersaturation of the urine with calcium oxalate to cause the precipitation of kidney stones. More than 80% of urinary oxalate is derived from endogenous production, primarily as a breakdown product of glyoxylate. The remainder of urinary oxalate is obtained from dietary sources. Therefore, hyperoxaluria can be caused by primary overproduction, intestinal disease, and diet. Overproduction of oxalate (primary hyperoxaluria) is hereditary and severe, but rare. Injury of the bowel wall inflicted by fatty acids or bile salts can result in an increased permeability to oxalate. The most usual clinical setting, as in this patient, is Crohn's disease, ileal resection, or jejunoileal bypass. A high dietary intake of oxalate may be due to the ingestion of foods such as chocolate, nuts, rhubarb, tea, and some fruit juices, as well as the intake of vitamin C in excess of 1,000 mg per day. Treatment usually involves the combination of a low-oxalate and low-fat diet together with administration of oral calcium or cholestyramine to bind oxalate in the intestine. Contrary to previously held notions, restriction of calcium intake could be deleterious.

Suggested Readings

Coe FL. The patient with renal stones. In: Schrier RW, ed. Manual of nephrology, 6th ed. Philadelphia: Lippincott Williams & Wilkins, 2005:90.

Monk RD, Bushinsky DA. Nephrolithiasis and nephrocalcinosis. In: Johnson R, Feehally J, eds. Comprehensive clinical nephrology, 2nd ed. Mosby, 2003.

Nephrotic Syndrome

  • What is the definition of nephrotic syndrome?

  • What are the causes of nephrotic syndrome?

  • What are the possible complications of nephrotic syndrome?

  • What are the treatment options for nephrotic syndrome?

Discussion

  • What is the definition of nephrotic syndrome?

    Nephrotic syndrome is a clinical entity characterized by (a) proteinuria in excess of 3.5 g/1.73 m2 of body surface area (or 50 mg/kg of body weight) per day; (b) hypoalbuminemia (<3 g/dL), which is a consequence of the renal losses

    P.413


    coupled with inadequate hepatic compensatory synthesis; (c) edema, which is a consequence of both the hypoalbuminemia and the sodium retention; (d) hyperlipidemia, which is probably due to the increased hepatic synthesis of very low-density lipoproteins which are converted to cholesterol-carrying low-density lipoproteins; and (e) presence of lipiduria. Impaired removal plays an important but probably secondary role in this setting.

  • What are the causes of nephrotic syndrome?

    The causes of nephrotic syndrome can be easily divided into two broad categories. The primary, or idiopathic, forms of nephrotic syndrome are those for which a specific cause cannot be identified despite a reasonably thorough evaluation. The five major histologic subtypes of primary nephrotic syndrome include minimal-change disease (also called lipoid nephrosis or nil disease), membranous glomerulonephritis, membranoproliferative glomerulonephritis (also called mesangiocapillary glomerulonephritis), focal segmental glomerular sclerosis (FSGS), and proliferative glomerulonephritis. The clinical and histologic characteristics of primary nephrotic syndrome are listed in Table 9-16.

    Table 9-16 The Clinical and Histologic Features of the Primary (Idiopathic) Nephrotic Syndrome

    Glomerular Disease Distinguishing Clinical and Laboratory Findings Characteristic Morphologic Features
    Minimal-change disease Most common cause in children (75%); 20% of adults; steroid- or cyclophosphamide-sensitive (80%); nonprogressive; normal renal function; scant hematuria LM: normal
    IF: negative
    EM: podocyte effacement; no immune deposits
    Focal segmental glomerulosclerosis Most common cause in adults (40%-50%); microscopic hematuria; progressive renal failure (75%) LM: early segmental sclerosis in some glomeruli with tubular atrophy; late sclerosis of most glomeruli
    Membranous nephropathy Peak incidence, fourth and sixth decades; male-female, 2-3:1; early hypertension (30%); spontaneous remission (20%); progressive renal failure (30%-40%) LM: early normal; late GBM thickening
    IF: granular IgG and C3
    EM: subepithelial deposits and GBM expansion
    Membranoproliferative glomerulonephritis Peak incidence, second through third decades; mixed nephrotic-nephritic features; slowly progressive in most, rapid in some; hypocomplementemia LM: hypercellular glomeruli with duplicated GBM
    EM: type I subendothelial immune deposits; type II dense deposit GBM
    Proliferative See Table 9-19  
    LM, light microscopy; IF, immunofluorescence; IgG, immu noglobulin G; EM, electron microscopy; GBM, glomerular basement membrane.

    P.414


    The secondary forms of the nephrotic syndrome are those associated with specific etiologic events or in which glomerular disease arises as a complication of another disease or systemic process. These may be broadly categorized into those stemming from infections, neoplasia, medications, allergens, multisystem diseases, and heredofamilial diseases, and also include various miscellaneous causes (Table 9-17). Secondary nephrotic syndrome may be associated with any of the major histologic subtypes found in idiopathic nephrotic syndrome. The idiopathic nephrotic syndrome is more common than the secondary form.

  • What are the possible complications of nephrotic syndrome?

    The complications of nephrotic syndrome include accelerated atherosclerosis, increased susceptibility to infections, osteomalacia, and an increased incidence of thromboembolic events.

  • What are the treatment options for nephrotic syndrome?

    The treatment of nephrotic syndrome depends on its cause. Certainly, in the case of the secondary nephrotic syndrome, if the primary disorder is treated effectively, the nephrotic syndrome tends to resolve as well. In the case of the primary nephrotic syndrome, certain histologic subtypes (i.e., minimal-change disease and possibly membranous nephropathy) respond to treatment with steroids, with or without cytotoxic agents. Discussion of the potential role for other agents such as cyclosporine or mycophenolate is beyond the scope of this book. Other lesions may be refractory to any type of therapy. Drugs such as the ACE inhibitors or ARBs may be useful in reducing the proteinuria by affecting intrarenal hemodynamics, but they cannot in any way alter the primary glomerular abnormality involved.

Case

A 40-year-old woman is referred for evaluation of proteinuria. Apart from occasional arthralgias, she has felt well but is concerned about progressive weight gain and marked swelling of her lower extremities. She has no personal or family history of renal disease, no known chronic systemic illness, nor is she taking any medications. Physical examination findings, including blood pressure, are normal, except for the presence of edema that is most notable in dependent areas. Laboratory evaluation reveals a normal hematocrit, as well as serum glucose, BUN, and creatinine levels, but she has profound hypoalbuminemia (1.9 g/dL) and hypercholesterolemia (490 mg/dL). Urinalysis shows 4+ proteinuria, oval fat bodies, and free fat droplets, but no cellular elements or casts. Her 24-hour urinary excretion of protein is found to be 8.6 g.

  • What is the most common cause of the secondary nephrotic syndrome in adults in the United States? In patients with this disorder, which early finding serves as a harbinger for the subsequent development of nephrotic syndrome and renal insufficiency?

  • What features of the history and physical examination are important in determining if this patient has a primary (idiopathic) or secondary form of the nephrotic syndrome?

  • What additional laboratory tests would you order either to establish or refute a secondary cause of the nephrotic syndrome?

  • How should this patient's evaluation proceed?

P.415


Table 9-17 Disorders Associated with Secondary Nephrotic Syndrome

  • Infectious diseases
    • Bacterial: poststreptococcal glomerulonephritis, infective endocarditis, nephritis, shunt syphilis, leprosy
    • Viral: hepatitis B and C, cytomegalovirus, Epstein-Barr virus, herpes zoster, human immunodeficiency virus infections
    • Protozoal: malaria, toxoplasmosis
    • Helminthic: schistosomiasis, trypanosomiasis, filariasis
  • Neoplastic diseases
    • Solid tumors (carcinoma and sarcoma): colon, lung, breast, stomach, kidney
    • Hematologic malignancies (leukemias and lymphomas)
  • Medications
    • Nonsteroidal antiinflammatory agents
    • Organic, inorganic, elemental mercury
    • Organic gold
    • Penicillamine
    • Street heroin
    • Probenecid
    • Bismuth
    • Captopril
  • Multisystem diseases
    • Systemic lupus erythematosus
    • Mixed connective tissue disease
    • Dermatomyositis
    • Dermatitis herpetiformis
    • Sarcoidosis
    • Henoch-Sch nlein purpura
    • Goodpasture's syndrome
    • Rheumatoid arthritis
    • Amyloidosis
    • Polyarteritis
  • Allergic reactions
    • Bee sting
    • Pollens
    • Poison ivy and poison oak
    • Serum sickness (antitoxins)
  • Metabolic diseases
    • Diabetes mellitus
    • Myxedema
    • Hyperthyroidism
  • Heredofamilial diseases
    • Alport's syndrome
    • Fabry's disease
    • Nail-patella syndrome
    • Sickle cell disease
    • a1-Antitrypsin deficiency
    • Congenital nephrotic syndrome (Finnish type)
    • Hereditary amyloidosis (familial Mediterranean fever)
  • Miscellaneous
    • Chronic renal allograft rejection
    • Pregnancy-associated (preeclampsia, recurrent or transient)
    • Vesicoureteric reflex

P.416


Case Discussion

  • What is the most common cause of the secondary nephrotic syndrome in adults in the United States? In patients with this disorder, which early finding serves as a harbinger for the subsequent development of nephrotic syndrome and renal insufficiency?

    Diabetes mellitus is the most common cause of secondary nephrotic syndrome in adults in the United States. In patients with either type 1 or type 2 diabetes, the onset of microalbuminuria (albumin excretion of 20 to 200 g per minute or 30 to 300 mg/g Cr per day) predicts the subsequent development of nephrotic syndrome and renal insufficiency. These patients should begin treatment with an ACE inhibitor or ARBs.

  • What features of the history and physical examination are important in determining if this patient has a primary (idiopathic) or secondary form of the nephrotic syndrome?

    Differentiating between the primary and secondary forms of the nephrotic syndrome depends on a careful review of the patient's history and physical examination findings and the performance of selected laboratory tests that can identify underlying disease states. It is imperative to determine if there is a family or personal history of diabetes mellitus or connective tissue disease, hereditary conditions such as sickle cell disease or Alport's syndrome, allergen exposure, and so forth. A complete medication list must be obtained, including the use of nonprescription medicines such as NSAIDs. A history of illicit drug use is equally important because heroin nephropathy is not rare in drug abusers. In addition, a travel history is a crucial part of the history taking because, for example, malaria is a well-known cause of the nephrotic syndrome and should be considered in those patients who have traveled to endemic areas. Risk factors for hepatitis and human immunodeficiency virus (HIV) infection must also be sought because high-risk populations should be screened for these disorders. In this particular patient (a young woman), the history of occasional arthralgias brings up the possibility of a multisystem disease as the source of the nephrotic syndrome.

  • What additional laboratory tests would you order either to establish or refute a secondary cause of the nephrotic syndrome?

    Laboratory tests that are useful in establishing a secondary cause of the nephrotic syndrome include the serum glucose level, an antinuclear antibody (ANA)

    P.417


    determination, complement levels, hepatitis screening, venereal disease research laboratory test, HIV test, sickle cell preparation, an antistreptolysin titer, throat culture, and serum and urinary protein electrophoresis. The findings yielded by the history and physical examination dictate which of these tests should be performed in a particular patient. In this patient, the ANA test is positive and the complement levels are low, indicating that she may have systemic lupus erythematosus (SLE) as the cause of her nephrotic syndrome.

  • How should this patient's evaluation proceed?

    In the setting of SLE, a kidney biopsy should be performed in an effort to establish the nature of the underlying disorder responsible for the nephrotic syndrome. This patient most likely has either diffuse proliferative glomerulonephritis or membranous nephropathy with SLE. The therapy for the former calls for treatment with steroids and cytotoxic agents, although the latter does not.

Suggested Readings

Bernard DB. Extrarenal complications of the nephrotic syndrome. Kidney Int 1988;33:1184.

Kaysen GA. Proteinuria and the nephrotic syndrome. In: Schrier RW, ed. Renal and electrolyte disorders, 6th ed. Philadelphia: Lippincott Williams & Wilkins, 2003:580.

Glomerulonephritis

  • What is the definition of hematuria?

  • What are the major causes of hematuria?

  • What can help point toward a glomerular origin as the source of the hematuria?

  • What is the definition of the nephritic syndrome?

  • What are the primary diseases of the kidney associated with glomerular hematuria (nephritic syndrome)?

  • What systemic diseases are associated with glomerular hematuria?

  • How is rapidly progressive glomerulonephritis (RPGN) defined?

  • What clinical disorders cause RPGN?

Discussion

  • What is the definition of hematuria?

    Hematuria refers to the presence of an abnormally high number of red blood cells (>5 per high-power field) in the urine. This is most commonly detected by a dipstick (Hemastix) method, which identifies the presence of hemoglobin. The hematuria is considered macroscopic when the urine is obviously red due to the presence of blood, and it is deemed microscopic when the urine grossly appears normal. A number of foods (such as beets) and some drugs (such as phenazopyridine hydrochloride) as well as porphyria can turn the urine red. In these circumstances, the dipstick result is negative.

  • P.418


  • What are the major causes of hematuria?

    The causes of hematuria are best approached in terms of their being either extrarenal or renal in origin. Extrarenal bleeding can occur in the ureters due to calculi or carcinoma; in the bladder due to hemorrhagic cystitis stemming from infection (including Schistosoma haematobium in endemic areas), as well as from cyclophosphamide use, carcinoma, catheterization, or calculi; in the prostate due to hypertrophy, carcinoma, or prostatitis; and in the urethra due to urethritis or trauma. Renal causes of hematuria can be classified as either glomerular or nonglomerular and are listed in Table 9-18.

  • What can help point toward a glomerular origin as the source of the hematuria?

    The following findings point toward a glomerular cause as the source of hematuria: (a) the presence of dysmorphic red blood cells on phase-contrast microscopy; (b) the presence of red blood cell casts, which is virtually a diagnostic finding; and (c) proteinuria exceeding 500 mg per day.

  • What is the definition of the nephritic syndrome?

    The nephritic syndrome is defined by a constellation of urinary findings that include the presence of hematuria, proteinuria, and red blood cell casts. These findings indicate the presence of a glomerular lesion and are frequently accompanied by azotemia, hypertension, and edema.

  • What are the primary diseases of the kidney associated with glomerular hematuria (nephritic syndrome)?

    The primary diseases associated with glomerular hematuria are immunoglobulin A (IgA) nephropathy, poststreptococcal glomerulonephritis, membranoproliferative glomerulonephritis, and idiopathic RPGN.

  • What systemic diseases are associated with glomerular hematuria?

    SLE, Henoch-Sch nlein purpura, Goodpasture's syndrome, vasculitis (including polyarteritis nodosa and Wegener's granulomatosis), and essential mixed cryoglobulinemia are all associated with glomerular hematuria.

  • How is RPGN defined?

    RPGN is primarily defined in clinical terms as a glomerular disease characterized by progression to end-stage renal disease within weeks to months. The pathologic correlate is extensive crescent formation in the glomeruli, as seen in kidney biopsy specimens.

  • What clinical disorders cause RPGN?

    A number of disorders cause RPGN. These are best defined in immunopathologic terms, depending on the absence or presence (and pattern) of immune deposits (Table 9-19).

Case

A 21-year-old college student is referred to the renal clinic for further evaluation of microscopic hematuria, which was discovered during a preemployment physical examination. There is no history of recent infections, trauma, or intravenous drug abuse. She denies any history of rashes, arthralgia, myalgias, fevers, or episodes of gross hematuria.

Physical examination reveals a well-developed, well-nourished woman who is in no acute distress. Her blood pressure is 125/85 mm Hg; pulse, 72 beats per minute; and

P.419


P.420


respiratory rate, 16 breaths per minute. No rashes, lymphadenopathy, or joint tenderness is noted. The remainder of the physical examination findings are within normal limits.

Table 9-18 Glomerular and Nonglomerular Renal Parenchymal Causes of Hematuria

  • Glomerular
    • Proliferative glomerulonephritis
      • Primary
      • Secondary
    • Familial diseases of the glomerulus
      • Alport's syndrome
      • Recurrent benign hematuria (thin basement membrane disease)
    • Malignant hypertension
  • Nonglomerular
    • Neoplasms
      • Renal cell carcinoma
      • Wilms' tumor
      • Benign cysts
    • Vascular
      • Renal infarct
      • Renal vein thrombosis
      • Malignant hypertension
      • Arteriovenous malformation
      • Capillary necrosis
      • Loin pain-hematuria syndrome
    • Metabolic
      • Hypercalciuria
      • Hyperuricosuria
    • Familial
      • Polycystic kidney disease
      • Medullary sponge kidney
    • Papillary necrosis
      • Analgesic abuse
      • Sickle cell disease and trait
      • Renal tuberculosis
      • Diabetes
      • Obstructive uropathy
    • Drugs
      • Anticoagulants (heparin, coumarin)
      • Drug-induced acute interstitial nephritis
    • Trauma
Adapted from Lieberthal W. Hematuria and the acute nephritic syndrome. In: Jacobson HR, Striker GE, Klahr S, eds. The principles and practice of nephrology. Philadelphia: BC Decker, 1991.

Table 9-19 Immunopathogenetic Classification of Rapidly Progressive Glomerulonephritis

  • Anti-GBM antibody (linear immune deposits)
    • With lung hemorrhage (Goodpasture's
    • Without lung hemorrhage (idiopathic)
  • Immune complex (granular immune deposits)
    • Predominantly IgA
      • IgA nephropathy
      • Henoch-Schonlein purpura
    • Predominantly IgG (others may be present)
      • Postinfectious
      • Visceral abscess
      • Bacterial endocarditis
      • Lupus nephritis
      • Cryoglobulinemia
      • Membranoproliferative glomerulonephritis
  • Pauciimmune (no immune deposits)
    • Vasculitis
      • Microscopic polyarteritis
      • Wegener's
      • Hypersensitivity vasculitides (e.g., Churg-Strauss syndrome)
    • Idiopathic
GBM, glomerular basement membrane; IgA, immunoglobulin A; IgG, immunoglobulin G.

The following laboratory data are reported: serum sodium, 135 mEq/L; potassium, 4.5 mEq/L; chloride, 105 mEq/L; carbon dioxide, 25 mEq/L; glucose, 98 mg/dL; BUN, 12 mg/dL; and creatinine, 0.8 mg/dL. Urinalysis shows a specific gravity of 1.015, pH of 5.0, 1+ heme, and 1+ protein on dipstick examination. Microscopic examination of the urine reveals 5 to 10 red blood cells per high-power field, and possibly one red blood cell cast is noted on close scrutiny of the entire slide. The 24-hour urine excretion is of 1.5 L total volume, with 1,200 mg of creatinine and 1,200 mg of protein.

On further laboratory examination, no secondary systemic cause for the nephritic syndrome is identified. Specifically, ANA and antineutrophil cytoplasmic antibody tests are negative, as are tests for hepatitis B and C. Likewise, both the C3 and C4 complement levels are normal. Consequently, a percutaneous renal biopsy is performed. The histologic, immunofluorescence, and electron microscopy findings are all consistent with IgA nephropathy.

  • What are the clinical entities that have been associated with prominent mesangial IgA deposits?

  • What clinical findings indicate a poor prognosis in IgA nephropathy?

  • What is the clinical course of IgA nephropathy?

  • P.421


  • What would you advise this patient if she were to contemplate pregnancy?

  • What treatment options are available for this patient?

Case Discussion

  • What are the clinical entities that have been associated with prominent mesangial IgA deposits?

    Henoch-Sch nlein purpura, chronic liver disease, dermatitis herpetiformis, axial arthropathies, and Berger's disease have all been found in the setting of mesangial IgA deposits.

  • What clinical findings indicate a poor prognosis in IgA nephropathy?

    The clinical findings that portend a poor prognosis in IgA nephropathy are persistent proteinuria of greater than 1 g per day, elevated blood pressure, male gender, an elevated serum creatinine level, and the absence of macroscopic hematuria.

  • What is the clinical course of IgA nephropathy?

    Patients with IgA nephropathy may experience intermittent episodes of gross hematuria, and 5% to 10% of the patients may have early nephrotic syndrome. End-stage renal disease develops in approximately 10% of affected patients by 10 years, and by 20 years in 20% of affected patients. In addition, another 20% to 30% may experience some decline in renal function within 20 years.

  • What would you advise this patient if she were to contemplate pregnancy?

    Despite early reports to the contrary, large retrospective surveys reveal no evidence indicating that IgA nephropathy unfavorably alters the course of pregnancy. In addition, the chances for a successful pregnancy are excellent if the patient remains free of hypertension or renal insufficiency.

  • What treatment options are available for this patient?

    There is no proven treatment for IgA nephropathy. The results of some trials of steroids have suggested that they are somewhat effective in patients with persistent proteinuria, when renal function is still well preserved (SCr <1.4 mg/dL).

Suggested Readings

Adler SG, Fairley K. The patient with hematuria, proteinuria, or both, and abnormal findings on urinary microscopy. In: Schrier RW, ed. Manual of nephrology, 6th ed. Philadelphia: Lippincott Williams & Wilkins, 2005: 116.

Glassock RJ. The glomerulopathies. In: Schrier RW, ed. Renal and electrolyte disorders, 6th ed. Philadelphia: Lippincott Williams & Wilkins, 2003:623.

Hyperkalemia

  • What are the causes of spurious hyperkalemia?

  • What are the primary mechanisms that underlie hyperkalemia, and what are the causes of each?

  • P.422


  • At what level of renal insufficiency does hyperkalemia occur?

  • What are the clinical consequences of hyperkalemia?

  • What therapeutic options are available for hyperkalemic patients, and how rapidly do they reverse the process?

Discussion

  • What are the causes of spurious hyperkalemia?

    The causes of spurious hyperkalemia (pseudohyperkalemia) comprise hemolysis of the blood sample, a marked leukocytosis (white blood cell count >50,000/mm3), thrombocytosis (platelet count >800,000/mm3), and an excessively tight tourniquet.

  • What are the primary mechanisms that underlie hyperkalemia, and what are the causes of each?

    The primary mechanisms that bring about hyperkalemia are an increased potassium input from either endogenous (e.g., hematomas or rhabdomyolysis) or exogenous sources, a transcellular redistribution of potassium, and decreased urinary excretion of potassium as occurs in renal insufficiency. The causes of these potassium-related abnormalities are listed in Table 9-20.

  • At what level of renal insufficiency does hyperkalemia occur?

    In the absence of other factors, hyperkalemia supervenes in patients with renal disease when the GFR is less than 10 mL per minute. The adaptive response to decreased renal mass involves the increased excretion of potassium per nephron; this maintains normokalemia despite an unchanged potassium intake (usually 60 to 80 mEq per day). However, in the presence of the processes listed in the answer to the previous question, hyperkalemia arises when the GFR is higher (as high as 40 mL per minute).

  • What are the clinical consequences of hyperkalemia?

    The most immediate and important impact of hyperkalemia is on the cells possessing excitable membranes (nerve and muscle) because it depolarizes such cells. The most significant effect of hyperkalemia is on the heart. The typical sequence of electrocardiographic changes seen with increasing degrees of hyperkalemia include tall, peaked T waves; P-wave abnormalities (including loss of the P wave); prolongation of the QRS complex; sinus arrest; atrioventricular dissociation; ventricular fibrillation; and cardiac arrest.

  • What therapeutic options are available for hyperkalemic patients, and how rapidly do they reverse the process?

    The various therapeutic options for hyperkalemia are listed in Fig. 9-2. As shown, calcium gluconate has the most rapid onset and should therefore be the first-line treatment to protect against the neuromuscular effects of hyperkalemia. Note also that the use of calcium gluconate, insulin with glucose, or sodium bicarbonate does not decrease total-body potassium content; unless a decrease in total-body potassium is achieved (e.g., with kaliuresis, kayexalate, or dialysis), hyperkalemia will recur when the therapeutic effect of these agents dissipates.

P.423


Table 9-20 The Causes of Hyperkalemia

  • Causes of increased potassium input
    • Exogenous potassium loads
      • Rapid intravenous potassium administration
      • High potassium intake with severe sodium restriction
    • Endogenous potassium loads
      • Rhabdomyolysis
      • Hemolysis
      • Tumor lysis syndrome
      • Hematomas
      • Increased catabolism
      • Burns
  • Causes of transcellular shift
    • Insulin deficiency
    • Metabolic acidosis due to mineral acid retention
    • Hypertonicity (glucose or mannitol)
    • Exercise
    • Hyperkalemic periodic paralysis
    • Digitalis intoxication
    • -Adrenergic antagonists
  • Causes of impaired renal excretion
    • Diffuse adrenal insufficiency (Addison's
    • Selective mineralocorticoid (aldosterone) deficiency
    • Primary renal tubular secretory defect
      • Obstructive uropathy
      • Sickle cell disease
      • Systemic lupus erythematosus
      • Renal transplantation
      • Tubulointerstitial nephropathy
    • Drug induced
      • Spironolactone
      • Triamterene
      • Amiloride
      • Inhibitors of the renin-angiotensin system
      • Pentamidine
      • Nonsteroidal antiinflammatory drugs
      • Calcineurin inhibitors
      • Trimethoprim
      • Heparin

P.424


Figure 9-2 Treatment of hyperkalemia. GFR, glomerular filtration rate; K, potassium. (From

Kelleher CL, Linas S. The patient with hypokalemia or hyperkalemia. In: Schrier RW, ed. Manual of Nephrology, 6th ed. Philadelphia: Lippincott Williams & Wilkins, 2005. Reprinted with permission.

)

Case

A 30-year-old white man has both diabetes mellitus and hypertension. The diabetes was diagnosed at 8 years of age when ketoacidosis developed. He has since had proliferative retinopathy, nephropathy, and peripheral and autonomic neuropathy. The nephropathy was recognized when the nephrotic syndrome developed 3 years ago, and there has also been a gradual increase in his serum creatinine level over the last 18 months. Hypertension was first detected a year ago. Although his serum glucose levels have in general been well controlled with the twice-daily administration of insulin, blood pressure control has been suboptimal despite treatment with losartan and hydrochlorothiazide.

The following physical examination findings are noted: supine heart rate of 76 beats per minute and blood pressure of 160/110 mm Hg; standing heart rate of 80 beats per minute and blood pressure of 130/90 mm Hg. Funduscopy reveals the presence of hemorrhages, exudates, and neovascularization. His lung fields are clear, no cardiac murmur is present, and there is trace lower extremity edema, decreased sensation to pinprick and vibration in the distal lower extremities, and absent deep tendon reflexes in the lower extremities.

The following laboratory values are reported: sodium, 138 mEq/L; potassium, 7.2 mEq/L; chloride, 110 mEq/L; carbon dioxide, 20 mEq/L; glucose, 129 mg/dL;

P.425


creatinine, 2.4 mg/dL; BUN, 30 mg/dL; and hemoglobin AIc, 8.8%. Electrocardiography demonstrates regular sinus rhythm at 76 beats per minute with a normal axis. The P waves are flattened, the QRS complex is 0.12 seconds in duration, and there are peaked T waves in the precordial leads. Urinalysis reveals a specific gravity of 1.015, pH of 5.0, 3+ protein, and hyaline casts. A 24-hour urine sample shows a creatinine clearance (CCr) of 35 mL per minute and 4.6 g of protein.

  • What do the electrocardiographic findings signify? How should the patient be treated?

  • What are the most likely factors contributing to this patient's hyperkalemia?

  • What are the drugs that can cause hypoaldosteronism?

  • What is the appropriate subsequent therapy for this patient?

Case Discussion

  • What do the electrocardiographic findings signify? How should the patient be treated?

    The electrocardiographic findings are characteristic of hyperkalemia. The patient should be treated immediately with calcium gluconate followed by measures to lower the serum potassium, as outlined in Fig. 9-2.

  • What are the most likely factors contributing to this patient's hyperkalemia?

    The major contributory factors responsible for the hyperkalemia in this patient include a decrement in the GFR, the use of losartan, and hyporeninemic hypoaldosteronism. Dietary potassium excess may be operant as well.

    The patient also has a metabolic acidosis that is probably contributing to the hyperkalemia. The development of hyperkalemia when the renal insufficiency is only moderate is likely because other factors are involved in the process. The syndrome of hyporeninemic hypoaldosteronism is common in patients with diabetes, and the presence of hyperchloremic acidosis further supports this possibility.

  • What are the drugs that can cause hypoaldosteronism?

    Inhibitors of the renin angiotensin system, heparin, NSAIDs, and spironolactone can all precipitate hypoaldosteronism. -Adrenergic blockers may contribute to hypoaldosteronism by impairing renin secretion. Spironolactone is a competitive inhibitor of aldosterone's cytosolic receptor, whereas amiloride inhibits potassium secretion through the operation of an aldosterone-independent mechanism. Calcium channel blockers have not been reported to inhibit aldosterone synthesis, but spironolactone is known to inhibit aldosterone action. Trimethoprim has been reported to have an amiloride-like effect in patients with the acquired immunodeficiency syndrome; pentamidine has similar effects in these patients. Calcineurin inhibitors also cause hyperkalemia, probably by an aldosterone-mediated mechanism.

  • What is the appropriate subsequent therapy for this patient?

    This patient should restrict his dietary potassium intake and take loop diuretics to manage the hyporeninemic hypoaldosteronism.

    His losartan (an ARB) dose needs to be decreased. Mineralocorticoid replacement can worsen the hypertension and sodium retention, and should therefore

    P.426


    be avoided. Sodium restriction should also be avoided because it attenuates the kaliuretic effect of the diuretic; sodium delivery is important to potassium excretion.

Suggested Readings

Kelleher CL, Linas S. The patient with hypokalemia or hyperkalemia. In: Schrier RW, ed. Manual of nephrology, 6th ed. Philadelphia: Lippincott Williams & Wilkins, 2005: 37.

Peterson LN, Levi M. Disorders of potassium metabolism. In: Schrier RW, ed. Renal and electrolyte disorders, 6th ed. Philadelphia: Lippincott Williams & Wilkins, 2003: 171.

Hyponatremia

  • What does the serum sodium concentration reflect, and what factors can alter the way in which it is interpreted? In what setting is pseudohyponatremia observed?

  • What is the underlying pathogenesis of hyponatremia?

  • What is the diagnostic approach to hyponatremia, and what are its major causes?

  • What are some of the drugs that produce hyponatremia?

  • What are the most common disorders associated with the syndrome of inappropriate antidiuretic hormone secretion (SIADH)?

Discussion

  • What does the serum sodium concentration reflect, and what factors can alter the way in which it is interpreted? In what setting is pseudohyponatremia observed?

    Hyponatremia represents a decrease in the concentration of sodium relative to that of water in the serum. Total-body sodium content may be decreased, unchanged, or even increased. The serum sodium concentration is a measure of the tonicity of body fluids, and it is the major contributor to the serum osmolality, as shown by the equation: Posm = 2 PNa + (glucose/18) + (urea/2.8), where Posm is the serum osmolality and PNa is the serum sodium concentration.

    Hyperglycemia can cause a decrement in the serum sodium level by shifting intracellular water out of cells. Because glucose is not freely movable across cell membranes, when the extracellular glucose concentration is elevated in insulin-deficient or -resistant patients, water moves out of the cells to equalize osmolality on both sides of the membrane. The movement of water dilutes the serum sodium concentration, but the serum osmolality is maintained. Clinically, hyperglycemia-induced hyponatremia is frequently encountered in the settings of diabetic ketoacidosis and nonketotic hyperosmolar coma. To determine whether a patient has a sodium or water deficit, the serum sodium level should

    P.427


    be estimated as if the patient were normoglycemic. The correction factor is as follows: for each 100-mg/dL increase in the serum glucose level, the serum sodium concentration decreases by 1.6 mEq/L. For example, if the sodium concentration is 109 mEq/L and the serum glucose content is 1,600 mg/dL, the corrected sodium concentration (Nac) would be calculated as follows:

    Therefore, the serum sodium concentration always needs to be interpreted in light of the glucose concentration. Events identical to these occur with exogenous mannitol administration.

    In pseudohyponatremia, the serum sodium concentration is low but the serum osmolality is normal. It occurs in settings of severe hyperlipidemia and hyperproteinemia, and is rare. The mechanism responsible for the low serum sodium concentration caused by hyperlipidemia and hyperproteinemia differs from that of hyperglycemia. At extremely elevated concentrations, both lipid and protein cause the sodium distribution space (i.e., plasma water space) to be decreased. Although the sodium concentration in plasma water is normal, it is decreased in the total plasma because of excess lipid or protein.

  • What is the underlying pathogenesis of hyponatremia?

    Hyponatremia arises when urinary dilution is abnormal. The ability to excrete a large volume of solute-free water depends on three factors: (a) normal fluid delivery to the distal nephron (i.e., normal GFR and normal proximal tubule reabsorption); (b) normal functioning of the thick ascending limb of Henle and the cortical diluting segments, which are sites of urinary dilution; and (c) the absence of vasopressin in the circulation, thereby allowing the collecting duct to remain water impermeable. In the presence of vasopressin, the tubular fluid equilibrates osmotically with the isotonic or hypertonic urine, thereby preventing the excretion of maximally dilute urine.

  • What is the diagnostic approach to hyponatremia, and what are its major causes?

    Once hyponatremia is confirmed, the next step is to determine whether it is associated with a low, normal, or high total-body sodium concentration. Usually, a physical examination can distinguish among these possibilities. Orthostatic hypotension and flat neck veins are seen in patients with a low total-body sodium content. Edema and ascites are common findings in patients with a high total-body sodium content. Patients with normal total-body sodium exhibit neither orthostatic changes nor edema. The major causes of each category of sodium concentration are summarized in Table 9-21.

  • What are some of the drugs that produce hyponatremia?

    Drugs can impair water excretion either by enhancing the renal action of vasopressin or by causing release of the hormone. Some of the more common agents are listed in Table 9-22.

  • P.428


  • What are the most common disorders associated with SIADH?

    In hospitalized patients, SIADH is the most common cause of hyponatremia. This is broadly due to a malignancy, pulmonary disorder, or central nervous system disorder, as shown in Table 9-23.

Table 9-21 Causes of Hyponatremia

Hypovolemia (Decreased Total-Body Sodium) Euvolemia (Near-Normal Total-Body Sodium) Hypervolemia (Increased Total-Body Sodium)
  • Extrarenal sodium losses
    • Vomiting (steady state)
    • Diarrhea
    • Fluid sequestration in
    • Peritonitis
    • Pancreatitis
    • Rhabdomyolysis
    • Burns
  • Renal sodium losses
  • Diuretics
  • Osmotic diuresis (glucose, urea, mannitol)
  • Mineralocorticoid deficiency
  • Salt-losing nephritis
  • Diuretics
  • Hypothyroidism
  • Glucocorticoid deficiency
  • Drugs
  • Pain or emotional stress
  • Respiratory failure
  • Positive-pressure breathing
  • Syndrome of inappropriate antidiuretic hormone secretion
  • Extrarenal disorders
    • Congestive heart failure
    • Hepatic cirrhosis
  • Renal disorders
    • Nephrotic syndrome
    • Acute renal failure
    • Chronic renal failure

Case

A 68-year-old man is hospitalized because of a persistent cough and 20-lb (9-kg) weight loss during the last 3 months. He has a 40-pack-year smoking history. On physical examination, he is found to be slightly confused and slow to respond. There are no orthostatic changes in his blood pressure or pulse. Chest examination reveals findings compatible with a left pleural effusion. Abdominal examination reveals no masses or organomegaly. There is no edema. He weighs 60 kg.

The following laboratory values are reported: sodium, 109 mEq/L; potassium, 3.4 mEq/L; chloride, 78 mEq/L; bicarbonate, 24 mEq/L; BUN, 4 mg/dL; glucose, 85 mg/dL; uric acid, 3.5 mg/dL; serum osmolality, 230 mOsm; and urine osmolality, 300 mOsm. A chest radiographic study shows a left pleural effusion. Purified protein derivative (PPD) testing is positive.

The patient's serum sodium concentration increases to 133 mEq/L within 24 hours. At that time, the patient is noted to be alert and his behavior appropriate. However, by the next day, he has become uncommunicative and agitated.

  • What are the most likely causes of hyponatremia in this patient, and why?

  • How do the serum potassium, BUN, and uric acid levels help in the assessment of this patient?

  • P.429


  • What are the primary considerations in treating patients with hyponatremia, and how should this patient's condition be managed?

    Table 9-22 Drugs Associated with Hyponatremia

    • Antidiuretic hormone analogs
      • Deamino-d-arginine vasopressin
      • Oxytocin
    • Drugs that enhance antidiuretic hormone release
      • Chlorpropamide
      • Clofibrate
      • Carbamazepine-oxcarbazepine
      • Vincristine
      • Nicotine
      • Narcotics ( -opioid receptors)
      • Antipsychotics or antidepressantsa
    • Drugs that potentiate renal action of antidiuretic hormone
      • Chlorpropamide
      • Cyclophosphamide
      • Nonsteroidal antiinflammatory drugs
      • Acetaminophen
    • Drugs that cause hyponatremia by unknown mechanisms
      • Haloperidol
      • Fluphenazine
      • Amitriptyline
      • Serotonin uptake inhibitors
      • Ecstacy (amphetamine related)
    aAntidiuretic hormone release may be secondary to underlying psychosis.
    From Berl T, Schrier RW. Disorders of water metabolism. In: Schrier RW, ed. Renal and electrolyte disorders, 6th ed. Philadelphia: Lippincott Williams & Wilkins, 2003:45. Reprinted with permission.

  • What could account for this patient's neurologic deterioration after his initial improvement?

Case Discussion

  • What are the most likely causes of hyponatremia in this patient, and why?

    This patient appears to have hyponatremia associated with a normal total-body sodium concentration because there are neither orthostatic changes nor edema. He therefore has euvolemic hyponatremia. Pituitary insufficiency appears clinically unlikely and no water-retaining medications are present, thereby making SIADH the most likely cause of the hyponatremia. The two leading diagnoses are lung cancer or pulmonary tuberculosis. In SIADH, a patient is slightly volume expanded. Therefore, as in this patient, the BUN and uric acid levels tend to be low. From the clinical

    P.430


    point of view, SIADH is the most likely diagnosis in this patient, but hypothyroidism should also be considered.

    Table 9-23 The Most Common Disorders Associated with the Syndrome of Inappropriate Secretion of Antidiuretic Hormone

    • Malignancy
      • Lung
      • Duodenum
      • Pancreas
      • Lymphoma
    • Pulmonary disorders
      • Pneumonia
      • Abscess
      • Aspergillosis
      • Respiratory failure
      • Positive-pressure breathing
    • Central nervous system disorders
      • Neoplasm
      • Encephalitis
      • Meningitis
      • Brain abscess
      • Head trauma
      • Guillain-Barr syndrome
      • Subdural or subarachnoid hemorrhage
      • Acute intermittent porphyria
      • Acute psychosis
      • Stroke
    • Other
      • AIDS
      • Prolonged exercise
      • Idiopathic (elderly)
    AIDS, acquired immunodeficiency syndrome.
    Modified from Berl T, Schrier RW. Disorders of water metabolism. In: Schrier RW, ed. Renal and electrolyte disorders, 6th ed. Philadelphia: Lippincott Williams & Wilkins, 2003:47. Reprinted with permission.

  • How do the serum potassium, BUN, and uric acid levels help in the assessment of this patient?

    The serum potassium concentration of 3.4 mEq/L and the BUN value of 5 mg/dL virtually rule out adrenal insufficiency because this is characterized by a hyperkalemic acidosis and an elevation in the BUN and serum creatinine levels as a consequence of volume contraction. Although the low serum potassium concentration brings into question the use of diuretics, the low uric acid level makes this unlikely. A low uric acid level is commonly observed in the setting of SIADH.

  • P.431


  • What are the primary considerations in treating patients with hyponatremia, and how should this patient's condition be managed?

    The optimal treatment for severe hyponatremia is still controversial because although profound hyponatremia is associated with high mortality and morbidity, its rapid correction may cause the formation of neurologic lesions, which are usually irreversible. The primary considerations in the therapy are the acuteness or chronicity of the process and the presence or absence of neurologic symptoms attributable to hyponatremia. The following are general treatment guidelines.

    In the setting of acute symptomatic hyponatremia with a change in mental status or seizures, the risk for complications stemming from cerebral edema exceeds the risk of complications due to rapid treatment. The patient should receive furosemide and hypertonic saline until convulsions subside.

    Asymptomatic hyponatremia is almost always chronic, and rapid correction is likely to do more harm than good. The treatment in these patients should consist of water restriction regardless of their serum sodium status.

    In the setting of symptomatic hyponatremia of chronic or unknown duration, the serum sodium level should be raised promptly by approximately 10 mEq/L through the administration of hypertonic saline, and then water restriction. A correction rate of 1 to 2 mEq/L per hour at any given time or an increase in the serum sodium level by more than 12 mEq per day should not be exceeded.

    In the present case, because the patient is symptomatic, it is prudent to correct the serum sodium level to approximately 120 mEq/L in 8 to 12 hours. The solute-free water loss needed to accomplish this may be estimated by multiplying total-body water (1 - actual serum sodium/desired serum sodium). Therefore, to correct the serum sodium in this 60-kg man from 109 to 120 mEq/L, he must have a negative water balance of 60 0.6 (1 - 109/120) = 3.3 L. This may be accomplished by infusing normal saline at a rate of 250 mL per hour while replacing urinary sodium losses with 3% saline so as to achieve a net solute-free water loss. A single injection of furosemide (20 mg IV) may be administered to promote diuresis; urinary potassium losses should be replaced. The serum sodium concentration may be raised by 1.0 to 1.5 mEq/L per hour. Once the serum sodium level has increased by approximately 10 mEq/L, this regimen should be discontinued.

    As for the long-term management of this patient, water restriction to 1,000 mL per day is the treatment of choice. However, because compliance may be difficult to achieve, demeclocycline can be given. This drug interferes with the antidiuretic hormone effect on the kidney and results in more dilute urine. If the patient's primary disease, lung cancer, or tuberculosis responds to treatment, this would likely promote resolution of the SIADH. Novel vasopressin antagonists that have aquaretic properties are likely to be preferable to demeclocycline.

  • What could account for this patient's neurologic deterioration after his initial improvement?

    This patient's serum sodium level increased by 24 mEq/L in the first 24 hours. This, therefore, puts him at risk for development of osmotic demyelination (OD) that is characterized by a flaccid quadriparesis, impaired speech and swallowing,

    P.432


    facial weakness, and poor response to painful stimuli. Pathologically, loss of myelin around nerve sheaths can be seen in pontine as well as extrapontine areas. The pathogenesis of this lesion remains unknown. There are several risk factors for the development of OD, including alcoholism, malnutrition, and burns, and it is also seen in women taking thiazide diuretics. The results of human and animal studies suggest that rapid correction of severe chronic hyponatremia may be associated with OD, whereas the hyponatremia itself is unrelated.

    The findings from studies of osmotically inactive solutes (such as amino acids, myoinositol, sorbitol, and methylamine) may have implications for the pathogenesis of OD. The intracellular levels of these solutes decrease slowly during the adaptation to changes in extracellular osmolality so that the cell volume is maintained. Therefore, in the setting of chronic hyponatremia, the rapid increase in extracellular osmolality may shrink brain cells, which have diminished osmotically active solutes as a consequence of adaptation to the chronic hyponatremia.

Suggested Readings

Berl T, Schrier RW. Disorders of water metabolism. In: Schrier RW, ed. Renal and electrolyte disorders, 6th ed. Philadelphia: Lippincott Williams & Wilkins, 2003:1.

Parikh C, Kumar S, Berl T. Disorders of water balance. In: Johnson R, Feehally J, eds. Comprehensive clinical nephrology, 2nd ed. Mosby, 2003.



Internal Medicine Casebook. Real Patients, Real Answers
The Internal Medicine Casebook: Real Patients, Real Answers
ISBN: 0781765293
EAN: 2147483647
Year: 2007
Pages: 14

flylib.com © 2008-2017.
If you may any questions please contact us: flylib@qtcs.net